57
BOARD QUESTIONS IN PHARMACOLOGY CHOOSE THE BEST ANSWER: MPL = 44.5 1. A 40 year old female who underwent total abdominal hysterectomy with bilateral salpingo-oophorectomy. To prevent osteoporosis what HRT regimen will be best for her. She claims to have no history of breast cancer in the family: A. Estrogen only preparation B. Estrogen and medroxyprogesterone acetate C. Medroxyprogesterone acetate D. Raloxifene MPL- 0.25 REFERENCE: PHARMACOLOGY 4 th Ed. Rang, pp. 440 2. The Patient at the delivery room is having uterine atony after giving birth to an 8-lb baby boy. Her BP at present is 150/90 mmHg. What agent should you give her: A. Methylergonovine maleate C. Carboprost B . Oxytocin D. Ergometrine MPL: 1 REFERENCE: PHARMACOLOGY 4 th Ed. Rang, et al pp.450 3. An old female was treated for endometriosis for 6 months. She claims to have been given a drug with androgenic effects such as hirsutism deepening of the voice and acne. Which of the following drugs could have been given: A. GnRH agonist C. Danazol B. Combined oral contraceptive pills D. Medroxyprogesterone acetate MPL: 0.25 REFERENCE: PHARMACOLOGY 4 th Ed. Rang, pp.447 4. A patient underwent hip replacement and is being given morphine for pain relief. At present her RR was noted to be at 8 cycles/min with prolonged episodes of apnea. What will you give in this patient? A. Atropine sulfate C. Naloxone B. Flumazenil D. Protamine sulfate MPL; 1 REFERENCE: PHARMACOLOGY 4 th Ed. Rang, pp.598 5. If you have a patient with seizure disorder which anesthetic agent will not be appropriate: A. Enflurane C. Halothane B. Nitrous oxide D. Procaine MPL: 0.25 REFERENCE: PHARMACOLOGY 4 th Ed. Rang, pp.523 6. A 50-year-old male with COPD is complaining of difficulty of sleeping for almost one month. Which of the following sedative hypnotics would be best for him: A. Diazepam C. Zolpidem B. Pentobarbital D. Thiopental MP: O.25 REFERENCE: PHARMACOLOGY 4 th Ed. Rang, pp.534 7. On waking up a patient who just underwent knee surgery cannot remember what happened while he is at the Operating Room. The anxiolytic agent that was probably given to him was _______. A. Buspirone C. Hydroxyzine B. Lorazepam D. Thiopental

Prc Board Questions in Pharma 2005

Embed Size (px)

DESCRIPTION

pharma

Citation preview

Page 1: Prc Board Questions in Pharma 2005

BOARD QUESTIONS IN PHARMACOLOGY

CHOOSE THE BEST ANSWER: MPL = 44.5

1. A 40 year old female who underwent total abdominal hysterectomy with bilateral salpingo-oophorectomy. To prevent osteoporosis what HRT regimen will be best for her. She claims to have no history of breast cancer in the family:

A. Estrogen only preparationB. Estrogen and medroxyprogesterone acetateC. Medroxyprogesterone acetateD. Raloxifene

MPL- 0.25 REFERENCE: PHARMACOLOGY 4th Ed. Rang, pp. 4402. The Patient at the delivery room is having uterine atony after giving birth to an 8-lb baby boy.

Her BP at present is 150/90 mmHg. What agent should you give her:A. Methylergonovine maleate C. Carboprost

B. Oxytocin D. ErgometrineMPL: 1 REFERENCE: PHARMACOLOGY 4th Ed. Rang, et al

pp.450 3. An old female was treated for endometriosis for 6 months. She claims to have been

given a drug with androgenic effects such as hirsutism deepening of the voice and acne. Which of the following drugs could have been given:

A. GnRH agonist C. DanazolB. Combined oral contraceptive pills D. Medroxyprogesterone acetate

MPL: 0.25 REFERENCE: PHARMACOLOGY 4th Ed. Rang, pp.4474. A patient underwent hip replacement and is being given morphine for pain relief. At

present her RR was noted to be at 8 cycles/min with prolonged episodes of apnea. What will you give in this patient?

A. Atropine sulfate C. NaloxoneB. Flumazenil D. Protamine sulfate

MPL; 1 REFERENCE: PHARMACOLOGY 4th Ed. Rang, pp.5985. If you have a patient with seizure disorder which anesthetic agent will not be appropriate:

A. Enflurane C. HalothaneB. Nitrous oxide D. Procaine

MPL: 0.25 REFERENCE: PHARMACOLOGY 4th Ed. Rang, pp.523

6. A 50-year-old male with COPD is complaining of difficulty of sleeping for almost one month. Which of the following sedative hypnotics would be best for him:

A. Diazepam C. ZolpidemB. Pentobarbital D. Thiopental

MP: O.25 REFERENCE: PHARMACOLOGY 4th Ed. Rang, pp.5347. On waking up a patient who just underwent knee surgery cannot remember what

happened while he is at the Operating Room. The anxiolytic agent that was probably given to him was _______.

A. Buspirone C. HydroxyzineB. Lorazepam D. Thiopental

MPL: 0.25 REFERENCE: PHARMACOLOGY 4th Ed. Rang, pp.5368. A 22-year-old female was found unconscious in her room. She presently broke up with

her boyfriend. By her bedside a number of sleeping pills and whiskey was noted. What could be the possible effect and interaction that occurred?

A. Additive C. PotentiationB. Antagonism D. Synergism

MPL: 0.33 9. If the patient will be given antidepressants. What adverse effect could be anticipated?

A. Insomnia C. Sedation B. Diarrhea D.TachycardiaMPL: 0.33 REFERENCE: PHARMACOLOGY 4th Ed. Rang, pp.55610. Antipsychotic potency generally runs parallel to the activity on which of the following

receptors:A. D1 B. D2 C. D3 D. D4

MPL: 0.25 REFERENCE: PHARMACOLOGY 4th Ed. Rang, pp.487

Next page pls.

Page 2: Prc Board Questions in Pharma 2005

- 2 –

11. The WHO classification of amphetamines:A. Cognition enhancer C. NeurolepticB. Hallucinogen D. Psychostimulants

MPL: 0.33 REFERENCE: PHARMACOLOGY 4th Ed. Rang, pp.46912. If the patient claims to have visual hallucinations, what could be the possible

psychotropic drug class that was taken?A. Antidepressant C. PsychotomimeticB. antipsychotic D. Psychomotor stimulant

MPL: 0.25 REFERENCE: PHARMACOLOGY 4th Ed. Rang, pp.46913. A student would like to improve her memory and cognitive performance. What class of

psychotropic drug will she choose:A. Cognitive enhancers C. PsychostimulantsB. Psychotomimetics D. Thymoleptics

MPL: 1 REFERENCE: PHARMACOLOGY 4th Ed. Rang, pp.46914. A bone cancer patient took high doses of morphine to relieve pain. What agent can be

used to reverse its toxic effects?A. Nalbuphine C. NalorphineB. Naloxone D. Methadone

MPL: 1 REFERENCE: PHARMACOLOGY 4th Ed. Rang,pp.59815. If respiratory depression would be the marked effect of the opioid analgesic, it is

probably more selective to what receptor subtype? A. Delta C. Kappa B. Gamma D. Mu

MPL: 0.25 REFERENCE: PHARMACOLOGY 4th Ed. Rang,pp.59216. Which of the following is not a manifestation of a patient taking an opioid analgesic.

Selectively interacting with mu receptors?A. Analgesia C. Physical dependenceB. Dysphoria D. Respiratory depression

MPL: 0.25 REFERENCE: PHARMACOLOGY 4th Ed. Rang,pp.592

17. If a child was brought to the clinic with skin abscess. Wound cotton done revealed the presence of Staph aureus. What antibiotic will be appropriate for this child:

A. Amoxicillin C. CloxacillinB. Cephalexin D. Vancomycin

MPL: 1 REFERENCE: PHARMACOLOGY 4th Ed. Rang,pp.693

18. The second generation cephalosporin which crosses the blood brain barrier:A. Cefoxime C. CefoxitinB. Cefuroxime D. Cefoperazone

MPL: 1 REFERENCE: PHARMACOLOGY 4th Ed. Rang,p.694

19. A G2P1 term, will be given prophylactic antibiotic prior to CS. Which of the following should be recommended:A. Cephalexin C. Cefazolin B. Cefoxitin D. Ceftriaxone

MPL: 0.3320. After prolonged treatment with penicillin, the patient developed pseudomembranous

colitis. What antibiotic will you give for this case:A. Aztreonam C. ImipenemB. Ceftriaxone D. Vancomycin

MPL: 0.25 REFERENCE: PHARMACOLOGY 4th Ed. Rang,p.70221. This agent inhibits topoisomerase II (DNA gyrase)

A. Clindamycin C. SpectinomycinB. Ciprofloxacin D. Tetracycline

MPL: 0.25 REFERENCE: PHARMACOLOGY 4th Ed. Rang,p.720

Next page pls.

- 3 –

Page 3: Prc Board Questions in Pharma 2005

22. In case of gram-negative septicemia if Ampicillin will be given with Gentamicin. What will be the expected response?

A There will be better chances of resolutionB. Similar effect with monotherapy C. Disease resolution will be delayed

D. No response will be notedMPL:1 REFERENCE: PHARMACOLOGY 4th Ed. Rang Pp.

69823.. A neonate was given IV antibiotics for 3 days. However, the baby had hypothermia,

diarrhea and other gray color. The baby was probably given:A. Amikacin C. ChloramphenicolB. Erythromycin D. Gentamycin

MPL: 1 REFERENCE: PHARMACOLOGY 4th Ed. Rang,p.692

24. The following antibiotics inhibit protein synthesis. Which of the following inhibits the translocation process:A. Erythromycin C. NetilmicinB. Chloramphenicol D. Tetracycline

MPL: 0.25 REFERENCE: PHARMACOLOGY 4th Ed. Rang,p.69925. The purpose of giving compound drug therapy in the treatment of tuberculosis:

A. to rapidly eradicate the strains of tubercle bacilliB. to shorten the infections phaseC. to decrease the emergence of resistant organismsD. to prevent complications of tuberculosis

MPL: 0.33 REFERENCE: PHARMACOLOGY 4th Ed. Rang,p.706

26. The patient on the 4th month of anti-TB treatment is having peripheral neuropathy. This may be attributed to which of the following:A. Isoniazid C. PyrazinamideB. Ethambutol D. Streptomycin

MPL: 1 REFERENCE: PHARMACOLOGY 4th Ed. Rang,p.704

27. Which of the following is NOT a first line agent in the treatment of tuberculosis?A. Ethambutol C. RifampicinB. Pyrazinamide D. Streptomycin

MPL: 1REFERENCE: PHARMACOLOGY 4th Ed.

Rang,p.70628. This agent is NOT included in the management if lepromatous leprosy:

A. Capreomycin C. DapsoneB. Clofazimine D. Rifampicin

MPL: 0.33 REFERENCE: PHARMACOLOGY 4th Ed. Rang,p.706

29. It is important for the clinician to avoid selection of an antiretroviral regimen that contains agents with similar toxicity profiles. Which regimen below contains at least 2 agents with similar toxicity profiles?A. Zidovudine, Lamivudine, Indinavir C. Zidovudine, Didanosine, SaquinavirB. Didanosine, Zalcitabine, Nelfinavir D. Stavudine, Lamivudine, Indinavir

MPL: 0.25 REFERENCE: PHARMACOLOGY 4th Ed. Rang,p.71430. A guanosine derivative which selectively inhibits viral DNA polymerase:

A. Aciclovir C. VidarabineB. Amantadine D. Zidovudine

MPL: 0.25 REFERENCE: PHARMACOLOGY 4th Ed. Rang,p.713

31. A patient taking Warfarin was also being treated for fungal infection. After 7 days, the patient developed epistaxis, Which of the following antifungal agents may have this interaction with Warfarin?A. Nystatin C. FlucytosineB. Ketoconazole D. Amphotericin B

MPL: O.25 REFERENCE: PHARMACOLOGY 4th Ed. Rang,p.317

Next page pls.

Page 4: Prc Board Questions in Pharma 2005

- 4 –

32. If the patient became anemia with neutropenia and thrombocytopenia during antifungal treatment. . What is the possible agent she is taking?A. Griseofulvin C. FlucytosineB. Fluconazole D. Terbinafine

MPL: 0.25 REFERENCE: PHARMACOLOGY 4th Ed. Rang,p.722

33. A7 year old patient was admitted because of pneumonia. On routine stool exam E. Histolytic Cyst 5 – 8/hpf was noted:

A. Anti- amebic treatment not necessaryB. Give diloxanide furoateC. Give metronidazoleD. Give both diloxanide furoate and metronidazole

MPL: 0.33 REFERENCE: PHARMACOLOGY 4th Ed. Rang,p.73634. Which of the following anti-malarial agent may promote radical cure?

A. Chloroquine C. PrimaquineB. Mefloquine D. Pyrimethamine

MPL: 0.25 REFERENCE: PHARMACOLOGY 4th Ed. Rang,p.72835. The drug of choice for mixed round worm infection:

A. Pyrantel pamoate C. NiclosamideB. Mebendazole D. Praziquantel

MPL: 0.50 REFERENCE: PHARMACOLOGY 4th Ed. Rang,p.740

36. On follow-up, after treatment with tapeworm infection, scolex was noted in the stool of the child:

A. Praziquantel C. Mebendazole

B. Niclosamide D. IvermectinMPL: 0.25 REFERENCE: PHARMACOLOGY 4th Ed. Rang,p.74237. Classified as anti-cancer antimetabolite agent:

A. cyclophosphamide C. MethotrexateB. Doxorubicin D. Paclitaxel

MPL: 0.25 REFERENCE: PHARMACOLOGY 4th Ed. Rang,p.67638. An agent used for cancer chemotherapy that inhibits purine synthesis:

A. Cytarabine C. MercaptopurineB. Fluorouracil D. Vincristine

MPL: 0.50 REFERENCE: PHARMACOLOGY 4th Ed. Rang,p.67739. Does not cause myelosuppression:

A. Bleomycin C. EtoposideB. Cisplatin D. Dactinomycin

MPL: 0.25 REFERENCE: PHARMACOLOGY 4th Ed. Rang,p.678

40. In a prescription made by an internist, the generic name was enclosed in a parenthesis and written below the brand name. This will be interpreted by the drug store as:

A. Violate prescription C. Erroneous prescriptionB. Impossible prescription D. Correct prescription

MPL: 0.50 Generics Act of the Philippines41. If the physician makes a prescription order that utilizes a drug supplied by the

pharmaceutical company, what class of prescription order was made:A. Extemporaneous C. PrecompoundedB. Compounded D. Erroneous

MPL: 0.2542. The response of a patient who has bronchial asthma on the following drugs maybe

graded as follows:A. Epinephrine > Norepinephrine > IsoproterenolB. Isoproterenol > Epinephrine >> NorepinephrineC. Isoproterenol > Epinephrine = Norepinephrine

D. Isoproterenol = Epinephrine >> Norepinephrine

Page 5: Prc Board Questions in Pharma 2005

MPL: 0.33 REFERENCE: PHARMACOLOGY 4th Ed. Rang,p.160

Next page pls.

- 5 -

43. A patient with septic shock was noted to have absent urine output for the past 12 hours. Which of the following drugs would be most helpful to this patient?

A. Furosemide C. DopamineB. Norepinephrine D. Epinephrine

MPL: 0.33 REFERENCE: PHARMACOLOGY 4th Ed. Rang,p.28844. A 40 year old male was seen at the ER because of anaphylaxis and was immediately

given Epinephrine. Apparently, The patient has taken prazosin for his hypertension. Which of the following maybe observed in this patient?

A. The patient may develop hypertensive crisis. B. There will be no effect on the patient’s blood pressure.

C. The patient may develop hypotension. D. The patient may develop severe difficulty of breathing due to bronchoconstriction.MPL: 0.33 REFERENCE: PHARMACOLOGY 4th Ed. Rang,P.152

45. Which of the following actions of norepinephrine is blocked by prazosin?A. Bronchial smooth muscle relaxationB. Vascular smooth muscle contractionB. Renin releaseC. Increased heart rate

MPL: 0.25 REFERENCE: PHARMACOLOGY 4th Ed. Rang,p.15246. Bradycardia maybe noted as reflex response when a patient is given

A. Dobutamine C. Norepinephrine B. Propranolol D. Methyldopa

MPL: 0.25 REFERENCE: PHARMACOLOGY 4th Ed. Rang,p.12747. Rifampicin and INH are given together for the treatment of pulmonary tuberculosis

because their drug interaction isA. Additive C. PotentiationB. Synergism D. Antagonism

MPL: 0.33 REFERENCE: PHARMACOLOGY 4th Ed. Rang,p.70348. If the plasma concentration of a drug declines with first order kinetic, this means that

A. there is only one metabolic path for drug disposalB. the rate of elimination is proportional to the remaining concentration

C. the drug is largely metabolized in the liver after oral absorption and has low bioavailability

D. the drug is not distributed outside the vascular systemsMPL: 0.50 REFERENCE: PHARMACOLOGY 4th Ed. Rang,p.8749. The following is NOT phase II drug metabolized reaction:

A. Oxidation C. MethylationB. Glucoronidation D. Acetylation

MPL: 0.25 REFERENCE: PHARMACOLOGY 4th Ed. Rang,p.7950. Mr. Jose Vida was admitted due to severe pneumonia. He was given a drug with a

volume of distribution of 40L and clearance of 80ml/min. What should be the loading dose in order to achieve a therapeutic plasma concentration of 40mg/L?

A. 0.1 mg/L C. 115.2 mg

Page 6: Prc Board Questions in Pharma 2005

B. 10 mg D. 160 mgMPL: 0.25 REF. BASIC & CLINICAL PHARMACOLOGY 9th Ed. KATZUNGp.4651. Two drugs A and B have the same mechanism of action. Drug A at a dose of 5 mg

produce the same magnitude of effect as drug B at a dose of 500 mg. This means thatA. Drug B is less efficacious than drug AB. Drug A is 100x more potent than drug BC. toxicity of drug A is less than that of drug B

D. Drug A is more effective than drug B.MPL: 0.50 REFERENCE: PHARMACOLOGY 4th Ed. Rang,p.29

Next page pls.

- 6 -

A 67 year old male was seen at the OPD due to epistaxis which occurred thrice for the past            week and easy bruising. He also  complained of abdominal discomfort and lack of appetite. PMH: Atrial fibrillation, Nocturnal heartburn Social history: Chronic alcohol abuse Medications: Warfarin, Digoxin, Cimetidine, Procainamide Pertinent laboratory exam: INR – 4.5

52. Which of the following could have contributed to the INR result of the patient?A. Cimetidine C. Chronic alcoholismB. Procainamide D. Digoxin

MPL: 0.50 REFERENCE: PHARMACOLOGY 4th Ed. Rang,p.374

53. The INR result is brought about by:A. Increased thrombin activityB. Decreased platelet activationC. Diminished levels of vitamin K

D. Decreased levels of factors II, VII. IX and XMPL: 1 REFERENCE: PHARMACOLOGY 4th Ed. Rang,p.31654. In the event this patient develop massive bleeding, which of the following is the best

thing to be done?A. Discontinue the drug and give vitamin KB. Discontinue the drug and change to LMWH.

C. Discontinue the drug and give protamine sulfate D. Discontinue the drug and transfuse platelets.MPL: 0.33 REFERENCE: PHARMACOLOGY 4th Ed. Rang,p.31955. If this patient’s ventricular rate cannot be controlled with digoxin, Which of the

following maybe used as an alternative for ventricular rate control?A. Amlodipine C. Quinidine

B. Atenolol D. IbutilideMPL: 0.25 REFERENCE: PHARMACOLOGY 4th Ed. Rang,p.26956. Which of the following is an adverse effect of procainamide?

A. Hypertension C. drug induced Lupus erythematosusB. Bradycardia D. Corneal microdeposits

MPL: 0.25 REFERENCE: PHARMACOLOGY 4th Ed. Rang p.229

Page 7: Prc Board Questions in Pharma 2005

A 26 year old medical student was brought to the ER for an apparent suicide poisoning due to broken heart. The maid could not recall the medication but claimed that the bottle contained medicine for headache. On PE, the physician noted cyanosis of the oral mucosa, tongue and nail beds.

57. Which of the following is the most likely drug ingested?A. Aspirin C. FlurbiprofenB. Acetaminophen D. Meloxicam

MPL: 0.2558. This DMARD agent is also an immunosuppressant.

A. Gold C. ChloroquineB. Penicillamine D. Cyclosporine

MPL: 0.50 REFERENCE: PHARMACOLOGY 4th Ed. Rang p.237

59. A 52 year old woman was suffering from severe joint pains and was diagnosed to have RA. However, she also appears to be suffering from acid peptic disease. Among the following, which is the safest to give her?

A. Aspirin C. CelecoxibB. Ibuprofen D. Phenylbutazone

MPL: 0.50

Next page pls.

- 7 -

60. Which of the following NSAID has the longest half-life and therefore should be given at a longer dosage interval?

A. meloxicam (20h) C. Tenoxicam (72h)

B. Rofecoxib (17h) D. Piroxicam (50-60h)MPL: 1 REFERENCE: PHARMACOLOGY 4th Ed. Rang p.230

A 21 year old male after quarrelling with his girlfriend took a bottle of insecticide from their garden approximately 30 –40 cc. He went back to his room and talked with his girlfriend over the phone. While talking to his girlfriend, he complained of dizziness, blurring of vision, generalized body weakness, numbness, severe epigastric pain and shortness of breath. He was able to call the attention of his older sister before falling on the floor. He was found lying on the floor and was noted to be pale, drowsy, with stiffening of the trunk and extremities and soaked with sweat. He was then brought to the ER. On arrival, he has drooling of saliva with 3 episodes of white viscid material, non-projectile non-bile stained about 2 tbsp. per bout.

61. As an ER physician, which of the following are you going to give the patient?A. Pilocarpine C. AtropineB. Physostigmine D. Carbachol

MPL: 0.33 REFERENCE: PHARMACOLOGY 4th Ed. Rang p.122

62. A diabetic patient develops hyperlipedemia (elevated LDL, and triglycerides, normal HDL). Which of the following is NOT appropriate for the patient?

A. Colestipol C. lovastatinB. Gemfibrozil D. niacin

MPL: 0.25 REFERENCE: PHARMACOLOGY 4th Ed. Rang p.308

63 . The patient was noted to have an elevated triglycerides level after treatment with a hypolipidemic agent. This side effect could be secondary to which of the following?

A. Cholestyramine C. cerivastatinB. Clofibrate D. niacin

MPL: 0.25 REFERENCE: PHARMACOLOGY 4th Ed. Rang p.30764. Inhibits de novo synthesis of cholesterol by inhibiting HMG CoA reductase:

A. Atorvastatin C. gemfibrozilB. Colestipol D. niacin

MPL: 0.50 REFERENCE: PHARMACOLOGY 4th Ed. Rang p.305

Page 8: Prc Board Questions in Pharma 2005

65. The patient has been having intermittent episodes of dry cough. The  agent that may have probably caused this is ______________:

A. Captopril C. losartan B. candesartan D. telmisartanMPL: 1 REFERENCE: PHARMACOLOGY 4th Ed. Rang p.29266. A patient with tachyarrhythmia and hypertension would be given an  antihypertensive.

What agent would be the choice for this patient? A. Felodipine C. nifedipineB. isradipine D. verapamil

MPL: 0.25 REFERENCE: PHARMACOLOGY 4th Ed. Rang p.27567. -blockers are given with vasodilators due to the following reasons:

A. reduce renal sodium excretionB. reduce renin releaseC. decrease systemic vascular resistance

D. increase cardiac contractilityMPL: 0.50 REFERENCE: PHARMACOLOGY 4th Ed. Rang p.287

68. Which of the following conditions will NOT benefit from the use of VASODILATORS:

A. hypertension C. anginaB. migraine D. peripheral vascular disease

MPL: 0.50 REFERENCE: PHARMACOLOGY 4th Ed. Rang p.290

Next page pls.

- 8 -

69. .An asthmatic patient came to the ER because of generalized wheezing in the lungs. Urgent management was done. However after awhile hypertension was noted. Which of the following medications was probably given?

A. salbutamol C. terbutalineB. isoproterenol D. salmeterol

MPL: 0.50 REFERENCE: PHARMACOLOGY 4th Ed. Rang P.16070. .A flight attendant has been having allergic rhinitis. She comes to your clinic asking for

medication prescription 2 hours prior her to flight. What will be appropriate for her? A. diphenhydramine C. loratadineB. Chlorpheniramine D. promethazine

MPL: 0.50 REFERENCE: PHARMACOLOGY 4th Ed. Rang p.34571. It acts by inhibiting the immediate phase of asthma as it prevents mediator release from mast cells

A. cromolyn sodium C. montelukastB. beclomethasone D. zileuton

MPL: 0.33 REFERENCE: PHARMACOLOGY 4th Ed. Rang p.34772. An asthmatic patient maintained on oral salbutamol for 8 months, apparently had

infrequent asthma attacks for the first 6 months. However, at present she had increased frequency of asthma attacks. This could be secondary to:

A. inadequate dose C. hypoxemiaB. poor compliance D. tachyphylaxis

MPL: 0.33 REFERENCE: PHARMACOLOGY 4th Ed. Rang p.1673. Which of the following is NOT a pharmacologic action of Cortisone: A. anti-inflammatory C. promotes fetal lung maturation

B. immunosuppressant D. adrenal suppressionMPL: 0.33 REFERENCE: PHARMACOLOGY 4th Ed. Rang p.41674. A patient with severe persistent asthma was treated for 6 months now  with prednisone.

Which of the following would be the expected side effect?A. Cushingoid facie C. hyperkalemia

Page 9: Prc Board Questions in Pharma 2005

B. hypoglycemia D. hypotensionMPL: 0.50 REFERENCE: PHARMACOLOGY 4th Ed. Rang p.42375. If a patient after prolonged steroid treatment developed depression. What synthetic

corticosteroid agent is he taking? A. cortisone C. fludrocortisoneB. dexamethasone D. prednisone

MPL: 0.50 REFERENCE: PHARMACOLOGY 4th Ed. Rang p.552

FM, 40 year old male was seen at the clinic for check-up. He has bradykinesia, muscle rigidity, pill rolling movement, resting tremors and shuffling gait.76. The single most effective agent in the treatment og this disorder is

A. Levodopa C. CarbamazepineB. Baclofen D. Tacrine

MPL: 1 REFERENCE: PHARMACOLOGY 4th Ed. Rang p.50977. Which of the following is NOT TRUE about this drug? A. In clinical practice, it is almost always used in combination with peripherally

acting inhibitor of aromatic L-amino acid decarboxylase. B. This drug has a short half-life about 1 – 3 hr. C. Administration of this drug with meals delay its absorption.

D. This drug has very little side effect and can be withdrawn abruptly. MPL: 0.33 REFERENCE: PHARMACOLOGY 4th Ed.

Rang p.51078. Which of the following is a selective inhibitor of MAO-B A. Selegiline C. Tolcapone B. Amantadine D. Bromocriptine MPL; 0.50 REFERENCE: PHARMACOLOGY 4th Ed. Rang p.511

Next page pls.

- 9 -

GM, a 53 year old male was seen at the OPD for regular check-up. He had a history of type II DM for 5 years and hypothyroidism for 20 years. Medications: Levothyroxine, Glyburide

79.The patient had a series of laboratory examinations and was noted to have hyperlipedemia. Which of the following drugs can interfere with the absorption of levothyroxine?

A. Simvastatin C. GemfibrozilB. Cholestyramine D. Niacin

MPL: 0.33 REFERENCE: PHARMACOLOGY 4th Ed. Rang p.70

LC, a 25 year old patient was seen at the clinic because of lump on her throat. She has gained 5 kg for the past 6 months, has cold intolerance, unusual painful heavy menses and constipation. She complained that the lump on her throat interferes with her swallowing.

PMH: iron deficiency anemia of 4 months durationMedications: Ferrous sulfate, Ibuprofen, Kelp tablets(contains Iodine), Lo-ovral (Ethinyl estradiol and norgestrel) daily

80.What effect does the use of oral contraceptive have on the patient’s hypothyroidism?A. Low dose oral contraceptives are unlikely to affect her test results.B. Estrogen may falsely increase TSH due to alterations in TSH secretionC. Estrogen may falsely decrease FT4I due to alterations in TBG.D. Estrogen may falsely elevate total T4 levels due to alterations in TBG.

MPL: 0.33 REF. BASIC & CLINICAL PHARMACOLOGY 9th Ed. KATZUNGp.630

Page 10: Prc Board Questions in Pharma 2005

81. If LC becomes pregnant, how might her levothyroxine replacement be affected? A. She may require a 20-30% increase in dose.

B. She may require a 20-30% decrease in dose. C. She will probably not require any changes in dose.

D. TT4 levels should also be monitored because of changes in TBG.MPL: 0.33 REF. BASIC & CLINICAL PHARMACOLOGY 9th Ed. KATZUNG p.631

A 62 year old male was seen at the ER because of lightheadedness, palpitation and shortness of breath. He said that the palpitations were associated with exercise that usually went away with rest. Two days ago, while washing the dishes, he began to have shortness of breath and felt that his heart was “racing”.

PMH: hypertension x 20 years; Hyperlipidemia for 5 years, and RHD with MVP as a child.Medications: Lisinopril, Furosemide, Gemfibrozil

82. Which of the following drugs is likely to be least effective in controlling this patient’s rapid ventricular response?

A Digoxin C. VerapamilB. Diltiazem D. Atenolol

MPL: 0.25 REFERENCE: PHARMACOLOGY 4th Ed. Rang p.26583. This patient was initially given verapamil for his AF. Due to the addition of this drug,

which of the following is the most appropriate treatment of this patient’s hypertension?A.Continue lisinopril and furosemide

B. Continue lisinopril and discontinue furosemide C. Continue lisinopril

D. Continue lisinopril, discontinue furosemide and add hydrochlorothiazideMPL: 0.25 REFERENCE: PHARMACOLOGY 4th Ed. Rang p.361

A 37 year old Fil-Am was seen at the clinic because of nausea and vomiting. Five days prior to consultation, he had nausea, vomiting, fever and chills. He took ibuprofen for generalized ache 3 days ago. Recent blood sugar reading was unavailable because he ran out of test strips . On admission, he was weak looking, severely dehydrated with weak pulses.

PMH: DM type I for 21 years, one episode of DKA 4 months ago; peripheral neuropathy

Medications: Insulin NPH< Ibuprofen, Amitryptilline

Next page pls.

- 10 -

84. Which of the following agents is least likely to aggravate the patient’s condition? A. Furosemide C. Acetaminophen

B. Amitryptilline D. InsulinMPL: 0.33 REFERENCE: PHARMACOLOGY 4th Ed. Rang p.36385. Loop diuretics have their principal diuretic effect on:

A. collecting ductsB. ascending limb*C. distal convoluted tubulesD. proximal convoluted tubules

MPL: 0.25 REFERENCE: PHARMACOLOGY 4th Ed. Rang p.36186. Which of the following are potential side effects of thiazide diuretics?       A. hypokalemia, hyperglycemia, hyperlipidemia

A. hypokalemia, ototoxicity, hyperuricemiaB. hyperkalemia, alkalosis, nausea/vomitingC. hyperglycemia, hypokalemia, metabolic alkalosis*

MPL: 0.33 REFERENCE: PHARMACOLOGY 4th Ed. Rang p.36487. Patients receiving insulin therapy generally require which of the following interventions

with progression to end-stage renal disease?

Page 11: Prc Board Questions in Pharma 2005

A. Decrease in total insulin dose C. Increase in frequency of insulin administrationB. Increase in total insulin dose D. No change in insulin regimen

MPL: 0.25 REF. BASIC & CLINICAL PHARMACOLOGY 9th Ed. KATZUNG p.694

BK a 23 year old female seen at the ER because of nausea and vomiting. Myalgia, polydipsia and polyuria. Three days prior to consult, he attended a party and drank an excessive amount of alcohol. He woke up sick to his stomach, vomited 6x since then and was unable to eat nor drink. He stopped taking insulin and currently has headache.

PMH: Type I DM for 11 years. Depression and allergic rhinitis Medications: Human insulin, Sertraline, Fluticasone, Loratidine, Acetaminophen

88. If Bk.’s blood glucose does not decrease by at least 2.8 mg/dl in the first 2 hours, what should be done?

A. Double insulin infusion rate C. Increase the insulin infusion to 0.5 /kg/hr

B. Give insulin bolus dose of 2U/kg D. Check blood glucose again in 2 hours

MPL: 0.5089. Which of the following statements about mixing NPH and regular insulins is NOT

TRUE?A. The injection must be taken within 10 mins of mixing

B. Regular insulin is drawn up before the NPH dose C. Mixtures of NPH and regular insulin are stable in any ratio

D. Mixing insulins allows greater flexibility versus 70/30 insulinMPL: 0.2590. Clostridium botulinum toxin produces respiratory paralysis by

A. blocking nicotinic receptors C. causing circulatory collapse

B. blocking the release of Ach from nerve endings D. stimulating vagusMPL; 0.25 REFERENCE: PHARMACOLOGY 4th Ed. Rang p.13091. A 60 year old male underwent cataract removal. After surgery Ach chloride was

administered intraocularly to:A. Relax the circular muscle of the iris C. decrease tearing from lachrymal secretion

B. ensure complete miosis D. decrease the flow of aqueous humorMPL; 0.50 REFERENCE: PHARMACOLOGY 4th Ed. Rang p.118

Next page pls.

- 11 -

92. An elderly patient was diagnosed to have open angle glaucoma. She was given 0.25% pilocarpine 2 gtts every 6 hours. The anticipated effect after the administration would be

A. relax ciliary muscleB. improve accommodationC. relax sphincter muscle of irisD. contract ciliary muscle and pull on trabecular network to relieve pressure

Page 12: Prc Board Questions in Pharma 2005

MPL: 0.33 REFERENCE: PHARMACOLOGY 4th Ed. Rang p.12193. Tachyphylaxis maybe observed with the use of the following:

A. Propranolol C. EphedrineB. Alpha methyl tyrosine D. Phenylephrine

MPL: 0.50 REFERENCE: PHARMACOLOGY 4th Ed. Rang pp.16, 15894. A 25 year old patient weighing 180 lbs consulted at the clinic. She tried several

methods of losing weight but failed. Her height is 170 cm. Based on her BMI she is considered:

A. healthy C. Obese B. overweight D. underweight

MPL: 0.33 REFERENCE: PHARMACOLOGY 4th Ed. Rangp.399

95. This inhibitor of GIT lipase is useful in the treatment of obesity and has minimal side effects:

A. sibutramine C. OrlistatB. Phentermine D. glucosan

MPL: 0.33 REFERENCE: PHARMACOLOGY 4th Ed. Rang p.40796. A group of students studied the effects of various GI drugs by measuring the effects on

gastric pH and volume. The baseline gastric volume was 2cc and when tested with litmus paper, the paper remained pink. After administering drug A, the gastric volume was 2 cc and the litmus paper turned blue. This means

A. The drug is effective in lowering gastric Ph B. The drug is effective in increasing gastric pH but no effect on the volume C. The drug is effective in lowering the gastric pH and volume D. The drug has no effect at all on volume and ph of gastric juice.MPL: 0.50 REFERENCE: PHARMACOLOGY 4th Ed. Rang p.37597. A patient taking this drug came into the clinic due to visual disturbances, constipation,

difficulty in urination and dry mouth. This drug could be:

A. muscarinic antagonist C. dopamine D2 antagonistB. histamine H2 antagonist D. gastrin antagonist

MPL: 0.50 REFERENCE: PHARMACOLOGY 4th Ed. Rang p.12298. The onset and duration of action of NPH insulin are extended because:

A. protamine decreases the rate at which insulin is absorbed B. protamine blocks insulin metabolism in the liver C. protamine is basic and combines with insulin by charge interactions D. protamine is slowly degraded proteolytically releasing the bound insulinMPL: 0.25 REF. BASIC & CLINICAL PHARMACOLOGY 9th Ed. KATZUNG p.69799. Which of the following is NOT TRUE about insulin action?

A. It stimulates glycogen synthesis in muscle fiber B. It inhibits lipolysis in the adipocyte C. It stimulates K fatty acid synthesis in the hepatocytes D. It stimulates gluconeogenesis in the hepatocytesMPL: 0.50 REFERENCE: PHARMACOLOGY 4th Ed. Rang p.387100. Which of the following drugs is most likely to improve gastric emptying in-patients

with diabetic gastroparesis? A. Loperamide C. magnesium hydroxide B. Cisapride D. Sucralfate

MPL: 0.50 REFERENCE: PHARMACOLOGY 4th Ed. Rang p.381

End ……………………………………………

Page 13: Prc Board Questions in Pharma 2005

Questions Answer Reference MPL

1. Extrapyramidal adverse effect is commonly seen with the use of this antipsychotic agent:

A. Haloperidol C. ChlorpromazineB. Thioridazine D. Fluoxetine

A Goodman and Gilman 10th ed p.488

1.0

2. An anti-malarial agent which also exerts an amebicidal effect

A. Chloroquine C. PrimaquineB. Chloroguanide D. Mefloquine

A Goodman and Gilman 10th ed p.1084

1.0

3. An anti-arrhythmic drug that predominantly block the slow calcium channels

A. Verapamil C. LidocaineB. Amiodarone D. Metoprolol

A Goodman and Gilman 10th ed p.949

0.25

4. Repeated exposure to which of the following anesthetic agents is most associated with liver damage

A. Isoflurane C. Nitrous oxideB. Halothane D. Enflurane

B Goodman and Gilman 10th ed p.351-352

0.33

5. The pharmacokinetic parameter which determines the speed of drug input that must balance the speed of drug elimination to achieve a steady state concentrationA. clearance C. dosing rateB. bioavailability D. volume of distribution

C Basic and Clinical Pharmacology 9th Ed. Katzung p.34-41

0.50

6. If a drug has a high distribution volume, it indicates that:A. it is highly bound to plasma proteinsB. drug is highly retained in the vascular compartmentC. the plasma drug concentration will increaseD. there is extensive tissue uptake

D Basic and Clinical Pharmacology 9th Ed. Katzung p.35 & 47

0.25

7. In what condition are the drugs phenoxybenzamine and phentolamine useful?

A. pheochromocytoma C. essential hypertensionB. migraine attacks D. benign prostatic hyperplasia

A Basic and Clinical Pharmacology 9th Ed. Katzung p.144-146

1.0

8. A college friend consulted you regarding the suitability of the therapy his doctor prescribed for severe

B Basic and Clinical

0.33

hypertension. He complained of postural and exercise hypotension (“dizziness”), diarrhea and problems with ejaculation. Which of the following is most likely to produce the effects that your friend has described?

A. Propanolol C. HydralazineB. Guanethidine D. Captopril

Pharmacology 9th Ed. Katzung p.166

9. Norpepinephrine acts as a neurotransmitter at:A. all preganglionic neuronsB. parasympathetic postganglionic neuronsC. sympathetic postganglionic neuronsD. neuromuscular junction

C Basic and Clinical Pharmacology 9th Ed. Katzung p.145

0.33

10. In the treatment of organophosphate poisoning, the mechanism of action of pralidoxime is:

A. competitive inhibition with acetylcholineB. non-competitive antagonism with acetylcholineC. regeneration of acetylcholineD. regeneration of acetylcholinesterase

D Basic and Clinical Pharmacology 9th Ed. Katzung p.103

0.33

11. A patient was given digitalis for heart failure, complained of nausea, vomiting, and diarrhea. PE revealed irregular cardiac rhythm. Which of the following is the most appropriate thing to do?

A. increase the dose of digitalisB. add a beta-blockerC. discontinue digitalisD. administer potassium

C Basic and Clinical Pharmacology 9th Ed. Katzung p.993

0.25

12. A 52 year old man consulted at the clinic due to hypertension (BP 190/90). Except for mild wheezing, his PE findings were unremarkable. The drug given can depress AV conduction, hence the use of beta-blocker is contraindicated. The drug that was given is:

A. Methyldopa C. Verapamil B. Enalapril D. Hydrochlorothiazide

C Basic and Clinical Pharmacology 9th Ed. Katzung p.235-236

0.33

13. A patient noted to have elevated cholesterol, LDL, and VLDL. Which among the following drugs act by blocking HMG-CoA reductase to inhibit cholesterol synthesis?

A. Probucol C. LovastatinB. Clofibrate D. Nicotinic acid

C Goodman and Gilman 8th ed p.881-884

1.0

Page 14: Prc Board Questions in Pharma 2005

14. A 20 year old female was treated for endometriosis for 6 months. She claims to have been given a drug with androgenic effects such as hirsutism, deepening of the voice, and acne. Which of the following drugs could have been given?

A. GnRH agonistB. Combined oral contraceptive pillsC. DanazolD. Medroxyprogesterone acetate

C Pharmaco-logy 4th ed page 447

0.25

15. A patient underwent hip replacement and was given morphine for pain relief. At present her RR was noted to be at 8 cycles/min with prolonged episodes of apnea. What will you give this patient to reverse the present condition?

A. atropine sulfate C. NaloxoneB. Flumazenil D. Protamine sulfate

C Pharmaco-logy 4th ed RangPage 598

1.0

16. A 7-year old was admitted because of pneumonia. On routine stool exam E. histolytica cyst 5-8/hpf was noted. What will you give?

A. Diloxanide furoate C. both A& BB. metronidazole D. no treatment necessary

A Pharmaco-logy 4th ed RangPage 736

0.33

17. A patient with septic shock was noted to have absent urine output for the past 12 hours. Which of the following drugs would be most helpful to this patient?

A. Furosemide C. DopamineB. Norpeinephrine D. Epinephrine

C Pharmaco-logy 4th ed RangPage 288

0.33

18. A 40 year old male was seen at the ER because of anaphylaxis immediately given epinephrine. Apparently, the patient has taken prazosin for his hypertension. Which of the following may be observed in this patient?

A. He may develop hypertensive crisis.B. There will be no effect on his blood pressureC. He may develop hypotension.D. he may develop severe difficulty of breathing due to

bronchoconstriction

C Pharmaco-logy 4th ed RangPage 152

0.33

19. Two drugs A and B have the same mechanism of action. Drug A at a dose of 5 mg produce the same magnitude of effect as drug B at a dose of 500 mg. This means that

A. drug B is less efficacious than drug A

B Pharmaco-logy 4th ed RangPage 29

0.50

B. drug A is 100x more potent than drug BC. toxicity of drug A is less than that of drug BD. drug A is more effective than drug B

20. A diabetic patient developed hyperlipedemia (elevated LDL, and triglycerides, normal HDL). Which of the following is least appropriate for the patient?

A. Colestipol C. LovastatinB. Gemfibrozil D. Niacin

D Pharmaco-logy 4th ed RangPage 308

0.25

21. A patient with tachyarrhythmia and hypertension would be given an antihypertensive. What agent would be the best choice for this patient?

A. Felodipine C. NifedipineB. Isradipine D. Verapamil

D Pharmaco-logy 4th ed RangPage 275

0.25

22. Which of the following conditions will NOT benefit from the use of vasodilators?

A. hypertension C. anginaB. migraine D. peripheral vascular disease

B Pharmaco-logy 4th ed RangPage 290

0.50

23. A patient at the delivery room is having uterine atony after giving birth toan 8-lb baby boy. Her BP at present is 150/90 mmHg. What agent should you give her:

A. methyergonovine maleateB. oxytocinC. carboprostD. ergometrine

B Pharmaco-logy 4th ed RangPage 450

1.0

24. The 2nd generation cephalosporin which crosses the blood brain barrier:

A. Cefoxime C. CefoxitinB. Cefuroxime D. Cefoperazone

B Pharmaco-logy 4th ed RangPage 694

1.0

25. A G2 P1 term, will be given prophylactic antibiotic prior to CS. Which of the following should be recommended?

A. Cephalexin C. CefazolinB. Cefoxitin D. Ceftriaxone

C Pharmaco-logy 4th ed RangPage 695

0.33

26. After prolonged treatment with penicillin, the patient developed pseudomembranous colitis. What antibiotic will you give for this case?

A. Aztreonam C. ImipenemB. Ceftriazone D. Vancomycin

D Pharmaco-logy 4th ed RangPage 702

0.25

27. This agent inhibits topoisomerase II (DNA gyrase)A. Clindamycin C. SpectinomycinB. Ciprofloxacin D. Tetracycline

B Pharmaco-logy 4th ed Rang

0.25

Page 15: Prc Board Questions in Pharma 2005

Page 72028. A neonate was given IV antibiotics for 3 days. However,

the baby developed hypothermia, diarrhea and grayish color of the skin. The baby was probably given:

A. Amikacin C. ChloramphenicolB. Erythromycin D. Gentamycin

C Pharmaco-logy 4th ed RangPage 692

1.0

29. The purpose of giving compound drug therapy in the treatment of tuberculosis:

A. to rapidly eradicate the strains of tubercle bacilliB. to shorten the infectious phaseC. to decrease the emergence of resistant organismsD. to prevent complications of tuberculosis

C Pharmaco-logy 4th ed RangPage 706

0.25

30. Which of the following is NOT a first line agent in the treatment of tuberculosis?

A. Ethambutol C. RifampicinB. Pyrazinamide D. Streptomycin

D Pharmaco-logy 4th ed RangPage 706

1.0

32.A patient taking warfarin was also being treated for fungal infection. After 7 days,the patient developed epistaxis. Which of the following antifungal agents was given? A.Nystatin C.Flucytosine B.Ketoconazole D.Amphotericin B

B Pharmaco-logy 4th ed RangPage 317

0.25

33.If the physician makes a prescription order that utilizes a drug supplied by the pharmaceutical company, what class of prescription order was made? A.Extemporaneous C.Precompounded B.Compounded D.Erroneous

C Generics Acts of the Philippines

0.25

34. Which among these antithrombotic drugs inhibit the synthesis of thromboxane by irreversible acetylation of the enzyme cyclooxygenase?A. Abciximab C. IntegrilinB. Aspirin D. Ticlopidine

B Pharmaco-logy Goodman and Gilman 10th ed. Pp 1534

0.50

35. A 60 year old was seen at the OPD due to epistaxis which occurred thrice for the past week and easy bruising. He is taking warfarin for AF. Perinent Lab exam: INR-4.5The INR result is brought about by which of the following? A.increased thrombin activity B.decreased platelet activity C.diminished levels of vitamin K D.decreased levels of factors II, VII,IX and X

D Pharmaco-logy 4th ed RangPage 316

1.0

36. A 67 year old with atrial fibrillation is currently taking A Pharmaco- 0.25

warfarin and digoxin. In the event this patient develops massive bleeding, which of the following would be appropriate for him? A.give vitamin K B.change to LMWH C.give protamine sulfate D.transfuse platelets

logy 4th ed RangPage 319

37.A 26 year old medical student was brought to the ER for an apparent suicide poisoning due to a broken heart. The maid could not recall the medication but claimed the bottle contained medicine for headache. On PE, the physician noted cyanosis of the oral mucosal, tongue and nail beds. Which of the following is the drug most likely ingested? A.Aspirin C.Flurbiprofen B.Acetaminophen D.Meloxicam

B Pharmaco-logy 4th ed RangPage 232

0.25

38.A 52 year old woman was suffering from severe joint pains and was diagnosed to have Rheumatoid Arthritis. Apparently, she also appears to be suffering from acid peptic disease. Among the following, which is the safest to give her? A.Aspirin C.Celecoxib B.Ibuprofen D.Phenylbutazone

C Pharmaco-logy 4th ed RangPage 234

0.50

39.A 21 year old male, took a bottle of insecticide from their garden approximately 30-40 cc in amount. He later developed dizziness, visual blurring, generalized body weakness, numbness, severe epigastric pain and shortness of breath. He was found lying on the floor with stiffening of the trunk and extremities and soaked with sweat. He was rushed to the ER, pertinent findings included drooling of saliva with 3 episodes of non projectile vomiting. Which of the following should be given? A.Pilocarpine C.Atropine D.Physostigmine D.Carbachol

C Pharmaco-logy 4th ed RangPage 122

0.25

40.This inhibits de novo synthesis of cholesterol by inhibiting HMG CoA reductase- A.Atorvastatin C.Gemfribozil B.Colestipol D.Niacin

A Pharmaco-logy 4th ed RangPage 305

0.50

41.An asthmatic patient came to the ER because of generalized wheezing in the lungs. Urgent management was done. However, after a while, hypertension was noted. Which of the following medications was probably given? A.salbutamol C.terbutaline

B Pharmaco-logy 4th ed RangPage 160

0.50

Page 16: Prc Board Questions in Pharma 2005

B.isoproterenol D.salmeterol42.A flight attendant has been having allergic rhinitis. She comes to your clinic asking for medication prescription two hours prior to her flight. What will be appropriate for her? A.diphenhydramine C.loratadine B.chlorpheniramine D.promethazine

C Pharmaco-logy 4th ed RangPage 345

0.50

43.This acts by inhibiting the immediate phase of asthma as it prevents mediator release from mast cells- A.cromolyn sodium C.montelukast B.beclomethasone D.zileuton

A Pharmaco-logy 4th ed RangPage 347

0.25

44.A patient with severe persistent asthma was treated for six months now with prednisone. Which of the following would be the expected side effect. A.cushingoid facie C.hyperkalemia B.hypoglycemia D.leonine facie

A Pharmaco-logy 4th ed RangPage 423

0.50

45.FM, 40 year old male was seen at the clinic for check-up. He has bradykinesia, muscle rigidity, pill rolling movement, resting tremors and shuffling gait. The single most effective agent in the treatment of this disorder is:

A. Levodopa C. CarbamazepineB. Baclofen D. Tacrine

A Pharmaco-logy 4th ed RangPage 509

1.0

46. Which of the following is a selective inhibitor of MAO-B?A. Selegeline C. TolcaponeB. Amantadine D. Bromocriptine

A Pharmaco-logy 4th ed RangPage 511

0.50

47. Which of the following statement is NOT true regarding Levodopa?

A. It is almost always used in combination with decarboxylase inhibitors.

B. It has a short half-life about 1 – 3 hrs.C. If administered with meals, its absorption is

delayed.D. It has very little side effect and can be withdrawn

abruptly.

D Pharmaco-logy 4th ed RangPage 510

0.25

48. What effect does the use of oral contraceptive (OC) have on a patient with hypothyroidism?

A. Low dose OC are unlikely to affect her thyroid function tests.

B. Estrogen may falsely increase TSH due to alteration in TSH secretion.

C. Estrogen may falsely decrease FT4 index due to

C Pharmaco-logy 4th ed RangPage 630

0.25

alterations in TBGD. Estrogen may falsely elevate total T4 levels due to

alterations in TBG49. If a hypothyroid patient pregnant, how might her levothyroxine replacement be affected?

A. She might require an increase in dose.B. She may need a decrease in dose.C. She may need to stop the medication.D. She may have to change levothyroxine to

liothyronine.

A Basic and clinical Pharmacology 9th Edition Katzung p 631

0.25

50. A 62 year old with atrial fibrillation was seen at the ER with light headedness and palpitations associated with exercise usually relieved by rest. At present his heart rate is at 140/min. Which of the following is least likely effective in controlling his heart rate?

A. Digoxin C. VerapamilB. Diltiazem D. Atenolol

A Pharmaco-logy 4th ed RangPage 265

0.25

51. Patients receiving insulin therapy generally require which of the following interventions with progression to end-stage renal disease?

A. decrease total insulin doseB. increase total insulin doseC. increase frequency of insulin administrationD. no change in insulin regimen

A Basic and clinical Pharmacology 9th Edition Katzung p 694

0.25

52. The inhibitor of GIT lipase is useful in the treatment of obesity and has minimal side effects:

A. sibutramine C. orlistatB. phentermine D. glucosan

C Pharmaco-logy 4th ed RangPage 407

0.25

53. An experiment on GI drugs was done. The baseline gastric volume 2 cc and when tested with litmus paper, The paper remained pink. After administering drug A, the gastric volume was 2 cc and litmus paper turned blue. This means

A. drug A is effective in lowering gastric pHB. drug A is effective in increasing gastric pH but no

effect on volumeC. The drug is effective in lowering the gastric pH and

volumeD. The drug has no effect on the volume and pH of

gastric juice

B Pharmaco-logy 4th ed RangPage 375

0.25

54.A patient taking this drug came into the clinic due to visual disturbances, constipation, difficulty in urination and dry

A Pharmaco-logy 4th ed

0.50

Page 17: Prc Board Questions in Pharma 2005

mouth. This drug could be: A.Muscarinic antagonist B.histamine H2 antagonist C.dopamine D2 antagonist D.gastrin antagonist

RangPage 122

55.The onset and duration of action of NPH insulin are extended because: A.protamine decreases the rate at which insulin is absorbed B.protamine blocks insulin metabolism in the liver C.protamine is basic and combines with insulin by charge interactions D.protamine is slowly degraded proteolytically releasing the bound insulin

B Basic and Clinical Pharmacology 9th Ed. Katzung p.697

0.25

56. An estrogen receptor antagonist which is useful for postmenopausal breast cancer

A. Megastrol C. FlutamideB. Tamoxifen D. Leuprolide

B Basic and Clinical Pharmacology 9th Ed. Katzung p.679

0.50

57. The agent of choice in the treatment of gestational diabetes

A. Sulfonylurea C. InsulinB. Biguanide D. Acarbose

C Pharmaco-logy 5th ed RangPage 388

1.0

58. A 40 year old bank executive sought consult due to a difficulty of going to sleep but no problem in maintaining sleep. He stopped drinking coffee, eating chocolates, and smoking a month ago. The most appropriate drug for him is:

A. Diphenhydramine C. PhenobarbitalB. Midazolam D. Clonazepam

B Basic and Clinical Pharmacology 9th Ed. Katzung p.362

0.33

59. A kidney transplant patient is given a drug for immunosuppression to prevent rejection of the allografted organ. The drug is most likely

A. Cyclosprine C. CyclophosphamideB. Azathioprine D. Vincristine

B Pharmaco-logy 5th ed RangPage 257

0.33

60. The preferred treatment for a patient who suffers a series without complete recovery is:

A. Diazepam C. SuccinylcholineB. Chlopromazine D. Ethosuximide

D Basic and Clinical Pharmacology 9th Ed. Katzung p.391

0.33

61. A company manager is suffering from allergic rhinitis. He has a series of staff meeting so he needs a medication that will not make him drowsy. Which of the following would you give?

A. Chlorpheniramine C. TerfenadineB. Hydroxyzine D. Diphenhydramine

C Basic and Clinical Pharmacology 9th Ed. Katzung p.266

0.50

62. A patient with severe liver disease eventually developed encephalopathy. Which agent is important in the therapy of this condition?

A. Lactulose C. LoperamideB. Lactate D. Lorazepam

A Pharmaco-logy 5th ed RangPage 375

1.0

63. A substance that enhances the probability of ovulation by blocking the inhibitory effect of estrogen and thus stimulating the release of gonadotrophin from the pituitary is:

A. Clomiphene citrate C. TamoxifenB. Danazol D. Progesterone

A Basic and Clinical Pharmacology 9th Ed. Katzung p.681

0.50

64. Which of the following combination (diuretic agent and MOA) is CORRECT?

A. Furosemide – inhibition of carbonic anhydraseB. Acetazolamide – osmotic effectC. Hydrochlorothiazide – inhibition of Na-Cl symportD. Indapamide – inhibition of Na-K-2Cl symport

C Basic and Clinical Pharmacology 9th Ed. Katzung p.249

0.50

65. Which of the following combination of diuretic and site of action on the nephron is CORRECT?

A. Ethacrynic acid – thick ascending limb of loop of Henle

B. Furosemide – proximal tubuleC. Hydrochlorothiazide – late distal tubuleD. Spironolactone – distal convoluted tubule

A Basic and Clinical Pharmacology 9th Ed. Katzung p.242

0.50

66. Which of the following is preferred for a pregnant woman with hyperthyroidism?

A. Carbimazole C. RAIB. Methimazole D. PTU

D Basic and Clinical Pharmacology 9th Ed. Katzung p.632

0.33

67. Drugs A and B act on the same tissue or organ through independent receptors, but with opposite effects. This is known as:

A. Competitive antagonismB. Irreversible antagonism

C Goodman and Gilman 8th ed p 53-54

0.5

Page 18: Prc Board Questions in Pharma 2005

C. Physiologic antagonismD. Chemical antagonism

68. This pharmacokinetic value most reliably reflects the amount of drug reaching the target tissue after oral administration

A. Volume of distributionB. Area under the blood concentration-time curveC. Peak blood concentrationD. Time to peak blood concentration

B Goodman and gilman 8th editionPp2627

0.5

69. A 3 year old patient passed out noodle-like worms and was given an appropriate anti-helminthic. The mechanism of the drug is most probably via

A. interference with cell wall synthesisB. inhibition of neuromuscular transmission

C. interference with cell wall divisionD. interference with protein synthesis

B Goodman and gilman 8th editionpp. 969-970

0.5

70. A patient with nosocomial pneumonia is given ceftazidime and another antibiotic. The most probabale purpose for adding the second antiobiotic is to:

A. Increase renal excretionB. Decrease systemic toxicityC. Increase oral absorptionD. Prevent the emergence of resistant bacteria

D Goodman and gilman 8th editionPp1127

1.0

71. About 75% of patients in this condition may suffer from vertigo, inability to perceive termination of movement and difficulty in sitting or standing without visual clues

A. Patients allergic to penicillinB. Patients receiving streptomycin therapyC. Patients on isoniazid for tuberculosisD. Patients on amphotericin B

B Goodman and gilman 8th editionPp1104 - 1108

0.5

72. A kidney transplant patient is given a drug for immunosuppression to prevent rejection of the allografted organ. The most appropriate drug is:

A. CyclosporineB. AzathioprineC. CyclophosphamideD. Vincristine

B Goodman and gilman 8th editionPp1270

0.5

73. A hypertensive patient is treated by a doctor and experiences hemolytic anemia with a positive Coomb’s test. This is most likely caused by:

A. Clonidine C. Methyldopa

C DiPalma 4th ed p. 472

0.50

B. Captopril D. Prazosin 74. In a hypertensive patient who is taking insulin to treat

diabetes, which of the following drugs should be used with caution?A. Propanolol C. Hydralazine B. Methyldopa D. Prazosin

A Goodman and Gilman 8th ed p.239

0.5

75. A patient has been taking Imipramine for depression. He would most likely experience which of the following adverse effects?A. Seizures C. hepatotxicityB. anticholinergic effects D. nephrotoxicity

B Goodman and Gilman 8th ed p.405-414

1.0

76. Chlorpromazine may be used not only in treating schizophrenia but is also effectiveA. in reducing nausea and vomitingB. as an antihistaminicC. as an antihypertensive agentD. for treating bipolar affective disorder

A DiPalma 4th ed p.275-277

0.5

77. A child suffers from lapses of consciousness or vacant stares lasting about 10 secs, often in clusters. The drug which may be most helpful for his disorder is:A. Phenytoin C. EthosuximideB. Carbamazepine D. Phenobarbital

C Goodman and Gilman 8th p. 449 – 453

0.5

78. Upon taking Penicillin, a patient suddenly experiences respiratory distress and circulatory collapse. Which drug would be most helpful in this situation?A. Norepinephrine C. IsoproterenolB. Epinephrine D. Atropine

B Goodman and Gilman 8th ed p.198

1.0

79. A gout patient developed nephropathy characterized by overproduction of uric acid and extreme hyperuricemia. What agent could have helped prevent this?A. Cyclophosphamide C. Sodium chlorideB. Allopurinol D. Antidiuretic hormone

B Goodman and Gilman 8th ed p.678

1.0

80. A patient with peptic ulcer and on therapy with cimetidine may experience decreased effects of many other drugs because cimetidine is an inhibitor ofA. the proton pump C. monoamine oxidaseB. tyrosine kinase D. cytochrome P-450

D Goodman and Gilman 8th ed p.901

1.0

73. The index of safety of a drug decreases as the median toxic dose:A. decreases and the median effective dose increases

A Basic and Clinical Pharmacol

0.33

Page 19: Prc Board Questions in Pharma 2005

B. increases and the median effective dose decreasesC. the median effective dose decreasesD. the median effective dose increases

ogy 9th Ed. Katzung p.30

74. What is the major second messenger of beta receptor activation that participates in signal transduction?A. inositol triphosphates C. calciumB. cAMP D. adenylyl cyclase

B Basic and Clinical Pharmacology 9th Ed. Katzung p.25-27

0.5

75. Which of the following bronchodilators is a xanthine derivative?A. Terbutaline C. theophyllineB. ipratropium bromide D. salmeterol

C Basic and Clinical Pharmacology 9th Ed. Katzung p.324, 325, 328

1.0

76. Myelosuppression is a common side effect associated with the use of the following anticancer drugs, EXCEPT:A. Vincristine C. MethotrexateB. Cyclophosphamide D. Chlorambucil

A Basic and Clinical Pharmacology 9th Ed. Katzung p.911

0.33

77. The following anticancer drug, with its toxic effect, is correctly matched with the appropriate antidote:A. cyclophosphamide-induced hemorrhagic cystitis –

ondansetronB. methotrexate-induced myelosuppression – MESNAC. cisplatin-induced nausea and vomiting – leucovorinD. doxorubicin-induced arrhythmia - dexrazoxane

D Basic and Clinical Pharmacology 9th Ed. Katzung p.914

0.50

78. Which of the following anticancer drugs act as a mitotic inhibitor?A. Methotrexate C. BleomycinB. Vinblastine D. Carmustine

B Basic and Clinical Pharmacology 9th Ed. Katzung p.911

0.75

79. Radical cure for malaria is achieved by giving:A. Chloroquine C. MefloquineB. Quinine D. Primaquine

D Basic and Clinical Pharmacology 9th Ed. Katzung p.871

1.0

80. A 3 year old was brought to the ER because of recurrent attacks of blank stares lasting for a few minutes. The anti-seizure agent that you will give the child is:A. Phenobarbital C. LamotrigineB. Ethosuximide D. Vigabantrine

B Goodman and Gilman 10th ed p.535

0.25

81. The mainstay drug in the treatment of all types of leprosy is:A. Clofazime C. IsoniazidB. Dapsone D. Rifampicin

B Goodman and Gilman 10th ed p.1288

1.0

82. A patient on anti-TB drug developed peripheral neuritis. This was improved by giving Pyridoxine. The drug taken must have been:A. Ethambutol C. IsoniazidB. Rifampicin D. Para-aminosalicylic acid

C Basic and Clinical Pharmacology 9th Ed. Katzung p.784

1.0

73. A patient with elevated cholesterol level is also taking warfarin for his deep venous thrombosis. The lipid lowering agent that may enhance the effect of Warfarin is:A. nicotinic acid C. cholestyramineB. lovastatin D. gemfibrozil

C Basic and Clinical Pharmacology 9th Ed. Katzung p.574

0.5

74. A known diabetic patient was brought to the ER unconscious with random blood sugar of 900 mg/dl. ABG study showed metabolic acidosis. Which of the following preparation is appropriate?A. regular insulinB. insulin zinc preparationC. isophane zinc suspensionD. potassium zinc insulin suspension

A Basic and Clinical Pharmacology 9th Ed. Katzung p.700

0.75

75. Which of the following corticosteroids has the greatest anti-inflammatory potency relative to cortisol?A. Prednisone C. AldosteroneB. Dexamehtasone D. Triamcinolone

B Basic and Clinical Pharmacology 9th Ed. Katzung p.647

0.5

76. A 30-year old G2P1 patient suffers experiences a regular and increasingly frequent contractions at 6 months of gestation. Which of the following agents may help in this case?A. Terbutaline C. Propanolol

A Basic and Clinical Pharmacology 9th Ed. Katzung

1.0

Page 20: Prc Board Questions in Pharma 2005

B. Nicotine D. Tacrine p.13877. The primary mechanism underlying the resistance of

gram positive organism to macrolide antibiotic is:A. methylation of binding site on the 50S ribosomal

subunitsB. formation of esterases that hydrolyzed the lactone

ringC. formation of drug-inactivating acetyltransferaseD. decrease drug permeability of the cytoplasmic

membrane

A Goodman and Gilman 10th ed p.1250

0.25

78. A prescription that has the generic name enclosed in a parenthesis and written below the brand name can be interpreted as:A. violative prescription B. imposiible prescriptionC. erroneous prescriptionD. correct prescription

C Generic Act of the Philippines

0.5

79. The onset and duration of action NPH insulin are extended by protamine because:A. it decreases the rate of insulin absorptionB. it blocks insulin metabolism in the liverC. it is a basic compound and combines with insulin

by charge interactionsD. it is slowly degraded proteolytically releasing the

bound insulin

B Pharmaco-logy 4th ed RangPage 697

0.25

80. The following anti-anginal drugs are vasodilators, EXCEPT:A. Felodipine C. MetoprololB. Nitroglycerin D. Diltiazem

C Basic and Clinical Pharmacology 9th Ed. Katzung p.186

0.50

81. Which is a direct effect of nitrates when given in the usual doses for the management of angina?A. increased preload B. increased afterloadC. increased cardiac contractilityD. decreased preload

D Basic and Clinical Pharmacology 9th Ed. Katzung p.190

0.50

73. The mechanism of action of OmeprazoleA. inhibits H+-K+ ATPase pumpB. promotes prostaglandin formationC. blocks action of histamine of H2 receptors

A Goodman and Gilman 8th ed p.902-904

1.0

D. neutralizes acid by increasing the pH of gastric secretion

1. 3rd generation cephalosporin that does not cross the BBBa. Cefoperazone (and Cefipime)b. Ceftazidimec. Moxolocamd. cefetamete. all of the above

2. associated with pseudomembronous colitis;a. methicillinb. Pen G Nac. Oxacillind. Ampicilline. Ticarcillin

3. Tetracyclines are contraindicated in children because it may causes;a. GI irritationb. Hepatotoxicityc. Bone and tooth discolorationd. Renal toxicitye. Only A and B

4. Anticonvulsants that have been shown to have antimanic effects;a. Carbamazepineb. Valproic acidc. Phenytoind. All of the abovee. A and B only

5. The major working hypothesis for schizophrenia, and the basis for much of drug therapy is the;

a. Serotonin hypothesis

Page 21: Prc Board Questions in Pharma 2005

b. Omine hypothesisc. Dopamine hypothesisd. Muscarinic hypothesise. Serotonim hypothesis

6. Causes harmless orange color of urine ,sweat and saliva;a. INHb. Rifampinc. Ethambutold. Cycloserinee. Ethionamide

7. This anticancer causes a minimal bone suppressant effect;a. Vincristineb. Methotrexatec. Cyclophosphamided. Cytarabinee. Doxorubucin

8. This is the drug of choice for mixed round worm infection;a. Mebendazoleb. Pyrantel pamoatec. Niclosomided. Thiobendazolee. Metronidazole

9. The fraction of the drug that reaches the general circulation is;a. Bioavalabilityb. Bioequivalencec. Biodispositiond. Biotransformatione. First pass

10. This deals with the biochemical and physiological effects of drugs and their mechanism of action

a. Pharmacokinetics

b. Pharmacodynamicsc. Pharmacotherapeuticsd. Pharmacogeneticse. Pharmacy

11. Which one of the following pairs of drugs has been known to cause a lupus like syndromea. Guarenthidine and reserpineb. Lidocaine and phenytoinc. Procainamide and hydralazined. Proprendol and disopyramide

12. Which one of the following statements is false about furosemide? Ita. Increases the excretion of Na, Cl, and waterb. Increases the excretion of Kc. Increases the excretion of uric acidd. May have to be used in conjunction with K sparing diuretic

13. The treatment of ketoacidosis may include the administration ofa. CaClb. Crystalline zinc insulinc. Glucagonsd. ZnCl

14. The action of cortisone is characterized bya. Causing hyperglycemiab. Decreasing gluconeogenesisc. Enchancing peripheral utilization of glucosed. Having an antiglucagon effect

15. Methimazole exerts its effects by inhibiting;a. Iodide acidificationb. Iodide transport and accumulationc. The formation of diiodothyrosined. The tubular secretion of iodine

16. Pharmacological properties of 5HT except:a. Enhances GIT motility

Page 22: Prc Board Questions in Pharma 2005

b. Increased force of contraction in the heartc. Bronchoconstrictiond. Bronchodilation

16. Piperazine derivatives excepta. Hydroxyzineb. Cetirizinec. Meclizined. Carbocisteine (an opioid)

17. They do not undergo the process of phosphorylation to exert their anti viral activitya. Indinavir (protease inhibitor)b. Cidofavirc. Epivird. Lamivudine

18. The following drugs reduce platelet aggregation by inhibiting the ADP pathwaya. Clopidogrel (& Ticlopidine)b. Abciximabc. Aspirin

19. Which drug decreases plasma triglycerides by increasing lipoprotein lipase;a. Lovastatinb. Nicotinic acidc. Cholestyramined. Probocole. clofibrate

20. Inhibits phosphodiesterase enzymea. Salbutamolb. Aminophyllinec. Cromolyn sodiumd. Ephedrine e. B & C

21. Agent of choice in congestive heart failure which are superior to other vasodilatorsa. Felodipine

b. Spironolactonec. Enalaprild. Digoxin

22. Occurs during phase 2 of action potential of Purkinje fibersa. Upstroke ends as Na channels are rapidly inactivatedb. K –channels rapidly open and close causing a transient outward currentc. Voltage-sensitive Ca-channels upon resulting in slow inward currentd. K-channels open resulting outward current

23. Orally given in patients with severe left ventricular systolic dysfunctiona. Digoxinb. Dobutaminec. Oraninoned. A and B only

24. An agent that neutralizes HCI secreted by parietal cells and increases gastric motilitya. Aluminum hydroxideb. Magnesium hydroxidec. Calcium carbonated. Sodium bicarbonatee. Calcium chloride

25. Contraindications for aspirin use except;a. Elderlyb. PUDc. Pregnancyd. Hemophiliae. Liver damage

26. A drug used in the treatment of gout that reduces uric acid synthesis by blockage of hypoxanthine;

a. Indomethacinb. Aspirinc. Colchicinesd. Allopurinol

Page 23: Prc Board Questions in Pharma 2005

e. Sulfinpyrazone27. The following is are True about Naftifine;

a. Interferes with ergosterol biosynthesisb. Inhibits squalene epoxidasec. Effects CYP 450 enzyme systemd. Significant drug interacte. A and B only

28. Inhibit DNA polymerase enzymesa. Didanosineb. Stavudinec. vidarabined. AZTe. A and C only

29. Specific 5HT3 antagonist used for nausea and vomiting secondary to chemo therapya. Methylgersideb. Ondansetronc. Metoclopromide

30. H1-receptor antagonist is which has anti-cholinergic properties excepta. Diphenhydramineb. Promethazinec. Meclizined. Estemizole

31. A patient was diagnosed as suffering from chronic refractory congestion and oliguria. Which one of the following agents is most effective?

a. Acetylcholineb. Atropinec. Dopamined. Terbutalinee. Propanolol

32. The nonselective beta-adrenergic blocking agent that is also a competitive alpha-adrenoceptors

a. Timololb. Madololc. Pindolold. Labetolole. Esmolol

33. All of the following compounds are direct-acting cholinergic agonist, excepta. Acetylcholineb. Pilocorpinec. Edrophoniumd. Bethanecol

34. Atropine and its analogs produce which of the following effects;a. Pupillary constrictionb. Increased gastric acid secretionc. Decreased secretion in the respiratory tractd. Increased peristalsise. Bronchoconstriction

35. The most potent of the inhalational anesthetics is;a. Enfluraneb. Halothanec. Isofluraned. Methoxyflurane

36. An ester local anesthetic with vasoconstrictive properties:a. Lidocaineb. Tetrocainec. Bupivacained. Cocainee. Procaine

37. Drug of choice for management of acid hypersecretion associated witha. Cholinergic agonistb. Prostaglandin analog

38. An antiarrhythmic drug that acts on K-channels

Page 24: Prc Board Questions in Pharma 2005

a. Esmololb. Sotololc. Diltiazemd. Adenosine

39. Prolongs phase 3 repolorizationa. Na-channel blockerb. B-adrenareceptor blockerc. K-channel blockerd. Ca-channel blocker

40. True statements about lidocaine excepta. Use for ventricular arrhythmias arising from MIb. Does not markedly slow conductionc. More effective on atrial or AV junction arrhythmiasd. None of the above

41. Calcium channel blocker which can relieve coronary spasma. Mifedipineb. Verapomilc. Diltiazemd. Hydralazine

42. Commonly used antihypertensive drugs in patients with insulin dependent diabetes mellitus

a. Calcium channel blockersb. ACE inhibitorsc. Both of the above

43. Effects of angiotensin II on adrenal cortexa. Stimulates catecholamine biosynthesisb. Stimulates mineralocorticoid synthesisc. Renal vasoconstriction

44. Specific for angiotensin AT receptorsa. Captoprilb. Candesertan

c. Voscosed. A and B only

45. Overuse of loop diuretics will cause which of the followinga. Hypokalemic metabolic alkalosisb. Hyperkalemic metabolic acidosisc. Hypokalemic metabolic acidosisd. Hyperkalemic metabolic alkalosis

46. The above condition is due to the secretion ofa. Sodium and hydrogen ionsb. Potassium and hydrogen ionsc. Sodium and chloride ionsd. Potassium and chloride ions

47. Which of the following should not be given to a hypertensivea. Lisinoprilb. verapamilc. methyldopad. Hydrochlorothiazide

48. Inhibits platelet aggregation by inhibiting ADP pathwaya. Aspirinb. Heparinc. Warfarind. Ticlopidine

49. Inhibits ligand binding to platelet GP IIb/IIIa receptor by occupancya. Abxicimabb. Tirofibanc. Enoxaparind. Anistreplase

50. Analog of aminocaprioc acid that competitively inhibits plasminogen activateda. Tranexamic acidb. Eprotinimc. Reteplase

Page 25: Prc Board Questions in Pharma 2005

d. Warfarin51. Effective as a singe drug therapy for familial dysbetalipoproteinemia

a. Fibric acidb. Bile acid-binding resinc. HMG-CoA reductased. Probucol

52. Most common side effect of antihyperlipidemic agentsa. Elevated blood pressureb. Neurological problemsc. Gastrointestinal disturbanced. Migraine headaches

53. Causes decrease in liver triacylglycerol synthesis by limiting available free fatty acid needed as building blocks for this pathway

a. Cholestyramineb. Elefibratec. Levostatind. Niacin

54. The following drugs maybe used during acute exacerbations of bronchial asthma excepta. Terbutalineb. Salmeterolc. Albuterold. Epinephrine

55. Effective as prophylactic anti-inflamatory agenta. Nedocromilb. Ipratropiumc. Theophyllined. Loratidine

56. Most reabsorption occurs at what segment of the nephrona. Distal convoluted tubuleb. Loop of Henlec. Collecting duct

d. Proximal convoluted tubule57. Are sulfonamide derived diuretic which inhibits Na+/CI- cotransporter at the DCT

a. Ethocrymic acidb. Acetazolormidec. Chlorthiazided. Triamtereme

58. Gynecomastia is an adverse effect of this diuretica. Mannitolb. Spironolactonec. Chlorthalidoned. Bumetinide

59. H2 receptor antagonist eliminated principally by the kidney hence has 100% bioavailability

a. Cimetidineb. Nizatidinec. Famotidined. Ranitidine

60. A target plasma theophylline concentration of 5mg/l is desired to relieve acute exacerbation of bronchial asthma in a 70 kg patient. If the mean clearance of this drug is 2800 ml/kg, what is the dose rate to be given via IV?

a. 7 mg/hb. 12mg/hc. 28 mg/hd. 14mg/h (2.8L/kg X 70kg)

61. Exacerbation was relieved and you want to maintain the plasma level rising oral theophyline every 12 hours using an extended release formula. If oral availlability is 96% what is the maintenance dose?

a. 175 mg/doseb. 350 mg/dosec. 700 mg/dosed. 700 mg/day

Page 26: Prc Board Questions in Pharma 2005

62. Single most important factor determining drug concentrationa. Clearanceb. Half-lifec. Absorptiond. Volume of distribution

63. A 15 year old maintained on phenobarbital was diagnosed to have typhoid fever. Which of the following antibiotic dose should be increased to achieve a therapeutic dose?

a. Trimethoprimb. Sulfamethoxazolec. Chloramphenicold. Ofloxacine. Paracetamol

64. MG. 38 year old female, 180 lbs, wanted to reduce weight by taking grapefruit juice. Which of the following drug metabolism will be inhibited?

a. Astorvastatinb. Cisapridec. Dicumerold. A and B onlye. All of the above

65. Active component of the above substance inactivatesa. Intestinal CYP3A4b. Hepatic CYP3A4c. Bothd. Neither

66. This/these refer/s to the concentration or those of drug required to produce 50 maximal effect

a. Potencyb. Efficacyc. Graded dose responsed. Quintal dose effect

67. Response to a drug diminishes rapidly after administration of a drug

a. Toleranceb. Tachyphylaxisc. Idiosyncracyd. Median effect

68. A synthetic derivative of glycyrrhizic acid that shows effective healing of gastric duodenal ulcers:

a. Sucrolfateb. Carbenaxolonec. Octreotided. Pirenzipine

69. Emulsifies with stool making passage easiera. Docusateb. Lactulosec. Bisacodyld. Phenolphthalein

70. Zileuton inhibits lipoxygenase pathway bya. Inhibiting 5-lipoxygenaseb. Antagonizing LTD4-receptorc. Alpha adrenoreceptor blockaded. Inhibiting phospholipase A2

71. Glucose transporter seen in muscle and adipose tissue which is responsible for intake uptake of glucose.

a. GLUT 4b. GLUT 3c. GLUT 2d. GLUT 1

72. Long acting insulin/sa. Lente humulinb. Insulin lisproc. Ultralente humulin U

73. Beta-lactam antibiotic of choice for enterobacter infections:

Page 27: Prc Board Questions in Pharma 2005

a. Tazobactomb. Meropenemc. Aztreonomd. Vancomycim

74. Tetracyclines maybe used to the following organisms excepta. Mycoplasma pneumoniaeb. Entameoeba histolyticac. Neisseria gonorrhead. Helicobacter pylori

75. Inhibits translocation by irreversibly binding to 50S ribosomal subunita. Azithromycinb. Gentamycinc. Fluroquinolone

76. A tissue Schizonticidea. Chloroquineb. Quininec. Mefloquined. Primaquine

77. Effective single agent for suppressing and curing multidrug resistant P. falciparuma. Quinineb. Mefloquine (???NOTA!)c. Pyrimethamined. Primaquine

78. Spectrum of activity of ribavirina. Rhinovirusesb. Enterovirusesc. Influenza virusesd. Hepatitis

79. Dideoxynucleoside that terminates the synthesis of proviral DNA chain and inhibits the RNA transcriptase of HIV and Hepatitis B virus

a. Stavudine

b. Lamivudinec. Zalcitabined. Saquinavir

80. Alkylating agents used as anticancer drugs excepta. Paclitaxel (Mitosis inhibitor)b. Cyclophosphamidec. Nitrosureasd. Mechlorethamine

81. A co-transmitter at inhibitory enteric nervous system neuromuscular junction that is important at sphincters:

a. GABAb. Golaninc. 5 HTd. NOe. CGRP

82. An autonomic receptor which results to opening of potassium channels and inhibits adenyl cyclase when activated

a. MIb. M2c. M3d. NNe. NM

83. An alpha adrenergic receptor blockera. Phentolamineb. Isoproterenolc. Norepinephrined. Betanechol

84. A direct acting cholinomimetic drug which is/are alkaloid or synthetic analoga. Carbonic acidb. Pilocarpinec. Carbachol

Page 28: Prc Board Questions in Pharma 2005

d. All of the above85. The drugs above causes

a. Cillary muscle relaxationb. Negative chronotrophyc. Positive chronotrophyd. Vasoconstriction at low dose

86. After a vehicular accident , the patient developed neurogenic bladder. A cholinomimetic commonly used in this case is

a. Betanecholb. Nicotinec. Muscarined. Neostigmine

87. Before giving the drug above you must make sure thata. There is no mechanical obstructionb. The level of cord injury is exactc. The patient had urinary retentiond. B and Ce. All of the above

88. A new direct acting muscarinic agonist used in the treatment of dry mouth associated with Sjogren syndrome

a. Embenoriumb. Denepezilc. Cevimeline (?di ko rin mabasa)d. Tacrinee. Pyridostigmine

89. A semisynthetic derivative of plant alkaloid podophyllatoxin blocking cells in the late S-G1 of cell cycle

a. Etoposideb. Procarbazinec. L-asparaginased. Cisplatin

90. True of aspirina. Reversible acetylators of cyclooxgenaseb. Depresses pain stimull at subcortical sitesc. Antinflammatory, antipyretic, antiplatelet, analgesicd. Responsible for keeping PDA open

91. An imidazopyridine derivative with hypnotic effects and facilitates GABA mediated inhibition

a. Zolpidemb. Buspironec. Zaleplond. Flumazeoil

92. Which of the ff. drugs is generally considered the drug of choice in treating status epilepticus

a. Phenoborbitalb. Amoborbital (Amstal)c. Phenytoin (dilantin)d. Paraldehydee. Diazepam (Valium)

93. The antihypertensive effect of guanethidine (Ismelin) is inhibited bya. Diazepam (Valium)b. Amitriptylinec. Hydrochlorothiazide (Hydro DIURIL)d. Probenecid (Benemid)e. Nitrofurantoin (furodentin)

94. A physician has decided upon a course of tetracycline therapy for a patient with renal impairment. Which of the following drugs is LEAST likely to accumulate in the blood

a. Demeclocyclineb. Doxycyclinec. Minocyclined. Oxytetracycline

Page 29: Prc Board Questions in Pharma 2005

95. Benztropine (Cogentin) is often given to patients taking the antipsychotic phenothiazine. Benztropine:

a. Reduces the doze of phenothiazine requiredb. Is an anticholinergic drug that reduces the extrapyramidal side effects of

phenothiazinec. Eliminates the unpleasant GI irritation caused by the phenothiazined. Is an antidepressant

e.reduces gut motility to ensure that the phenothiazine is completely absorb

92. The metabolism of which of the following compounds is altered in patients taking antimetabolitea. Pyridoxineb. Folic acidc. Riboflavind. Rennine. Tyroxine

93. Which of the following antifungal agents is ineffective against candida organism?a. nystatin (mycostatin) d. halopragin (halotex)b. clotrimazole (lotrimin) e. miconazolec. tolnaftate (tinactin)

94. The clinical investigation of a new drug consists of four phases. Phase I of the clinical testing involves administering the drug. a. to animals to determine side effects of the drug b. to animals for toxicity studies c. by select clinicians to healthy volunteers d. by select clinicians to patients suffering from the disease e. by general practitioners to patients suffering from the disease95. Tubocurarine should NOT be used in patients who are taking

a. aspirinb. morphinec. indomethacind. levodopae. gentamicin

96. Penicilloyl-polytysine is a substance used to: a. stabilized crystalline penicililin G preparations

b. counteract allergic reaction to penicillinc. reduce the renal secretion of penicillin

d. skin test patients for penicillin allergye. manufacture the semisynthethic penicillins

97. Ticarcillin may be preferred to carbenenicillin for patients with congestive heart failure, renal failure, and hypertension because it

a. contains less sodium than does corbenicillinb. is usually given in smaller doses than carbenicillinc. does not induce hypertensiond. stimulates renal blood flowe. has a broader spectrum than does carbenicillin

98. A Fanconi-like syndrome has been associated with the use of outdated and degraded a. ampicillin b. cloxycycline c. tetracycline d. chloromphenicol e. clindemycline99. Which of the following is true of lithium carbonate (Eskolith, Lithane) a. Indicated in the treatment of severe manic-depressive psychoses b. May only be administered by the intremuscular route c. Onset of action occurs within 2 hours of the first administered dose d. Should be administered with a diuretic to minimize edema formation100. Patients on lithium carbonate therapy should be advised a. to limit water intake b. to stop taking the drug if they experience mild side effects c. not to restrict their normal dietary salt intake d. not to take the drug during the manic phase of their cycle e. not to take the drug with food101. Which of the following drugs can interfere with the diagnosis of pernicious anemia

a. Pyridoxineb. Menadionec. Thiamined. Ascorbic Acide. Folic Acid

102. Which of the following agents would be most dangerous to use in a patient with high doses of gentamicin? a. ethacrymic acid (Edecrin) b. tetracycline HCI c. propantheline bromide (pro-banthine) d. hydroOIURIL e. pentobarbital sodium103. Diethylstilbestrol – induced vaginal adenocarcinoma is an example of a. Delayed irreversible toxicity b. dose – dependent toxicity

Page 30: Prc Board Questions in Pharma 2005

c. hypersensitivity reaction d. idiosyneratic reaction104. Ethylene glycol poisoning would lastly manifest as

a. acidosisb. alkalosisc. oxalate crystals in the urine d.renal insufficiency

105. Isopropyl alcohol poisoning resembles the poisoning produced by which one of a. ethyl alcohol b. ethylene glycol c. kerosene

d. methyl alcohol

102. Benzene toxicity is characterized by

a. anemia

b. otoxia

c. gastrointestinal symptoms

d. respiratory alkolocis

103. Immunnosuppressive agents such as prednisone and azothioprine have been..for all of the following disorders, except which one?

a. active hepatitis

b. megalobiastic anemia

c. thrombocytopenic purpura

104. The metabolite of azathioprine is

a. Cloflucarbon

b. 6-mercaptopurine

c. merodiceire

d. triclosamide

105. The chief side effect of azathioprine is

a. alopecia

b. leucopenia

c. hypertrichosis

106. The toxicity of trimethadione is manifested as which of the following

a. alopecia

b. bone narrow depression

c. hemerolopia

107. One reason nitrosoureas are effective in the treatment of brain tumors

a. are lipid soluble

b. chelate carbonium ions

c. chelate isocyanate molecules

d. inhibit the generation of carbonium ions

108. 5-Fluorouracil is given intravenously because it is

a. a gastric secretogogue

b. metabolized rapidly in the liver

c. not absorbed orally

d. a potent irritant to the gastric mucous

109. In brain tumors, which one of the following cells is deficient?

a. astrocytes

b. eosinophils

c. monocytes

d. T4- helper lymphocytes

110. Leprosy is treated mainly with

a. guenobenz

b. Guanfocine

c. methylphenidate

d. sulfoxone

111. Primaquine is characterized by the following characteristic, except which one ?

a. attacks plasmodia in exoerythrocytic stages

b. is effective for malarial prophylaxis

c. may cause hemolytic anemia in susceptible patients

d. may cause polycythemia vera

Page 31: Prc Board Questions in Pharma 2005

112. Thiabendozole is characterized by the which of the following

a. exerts its effects by inhibiting fumarate reductase

b. is metabolized by dehydration

113. Third generation cephalosporins such as cefotaxime and ceftazidime

a. are effective in treating infection caused by aerobic gram-negative

b. do not require dose adjustment in patients suffering from renal imparement

c. have identical pharmacokinetic properties

d. require dose adjustment in patients with hepatic diseases

114. All of the ff. chemotherapeutic agents have been properly matched with their characteristics except one, which one?

a. azlocillin – is less potent than carbenicillin

b. carbenicillin indanyl- is effective orally

c. mezlocillin – is effective in Klebsiella infection

d. ticarcillin – is more potent than carbenicillin

115. Which one of the following penicillin derivatives is not stable in gastric acid

a. amoxicillin

b. dicloxicillin

c. penicillin G

116. Chloramphenicol is characterized by all of the following actions, except which one?

a. bacterial resistance results from the development of chloramphenicol acetyltransferases

b. it has a narrow spectrum of activity

c . it is a drug of choise in the treatment of salmonella infection

d. it is used for treating meningococoal onfection in patients allergic to penicillin

118. A 30 year old overweight woman suffered from paroxysmal supraventricular tachycardia and also borderline hypertension (145/90mml-ig). For which she was not taking any medications. The cardiologist should treat the tachycardia with which one of the following

a. digitals followed by quinidine

b. propanolol

c. propanolol plus thiazide

d. verapamil

118. All of the following agents participate in secretion in the exocrine pancreas, except

a. acetylcholine

b. cholecystokinin

c. oxytocin

d. substance P

119. All of the following agents belong to the phenylalkylamine – derived calcium entry family except one, which one

a. gallopomil

b. oxodipine

c. ronipamil

e. verapamil

120. All of the ff. agents are dihydropyridine derived calcium entry blockers:

a. anipamil

b. nicardipine

c. nifedipine

d. riodine

121. Which one of the ff. is the least recommended indication for verapamil

a. atrial flutter

b. atrial fibrillation

c. digitalis toxicity

d. reentrant paroxysmat supreventricular

122. The pertussis toxin

a. alters the activity of adenylate cyclase

b. resembles botulinum toxin in action

c. resembles the cholera toxin in action

d. resembles forskolin in action

123. Activation of all of the ff. receptor sites causes stimulation of adenylate cyclase except one, which one?

Page 32: Prc Board Questions in Pharma 2005

a. alpha2- adrenergic receptors

b. beta- adrenenergic receptors

c. D1 dopaminergic receptors

d. serotonerg ic receptors

124. Which one of the ff. best describes the actions of morphine and enkephalin on adenylate cyclase

a. activate adenylate cyclase in therapeutic doses

b. inhibit adenylate cyclase in toxic doses

c. produce a selective and naloxone-reversible inhibition of adenylate cyclase

d. produce a nonspecific and naloxone-insensitive activation of adenylate cyclase

125. Which one of the ff. substances does not increase the concentration of cyclic AMP

a. aminophylline

b. epinephrine

c. glucagon

d. insulin

126. Renal cell carcinoma-induced hypercalcemia may be reduced by the administration of the ff. agents?

a. aspirin

b. calcitonin

c. parathyroid hormone

d. sodium citrate

126. The prostacyclin-induced secretion of renin is shared by which one of the ff.

a. acetylsalicylic acid

b. indomethacin

c. meclofenomate

e. norepinephrine

127. The bronchodilating action of prostaglandin E1 is shared by

a. acetylcholine

b. bradykinin

c. histamine

d. protoglandin E2

128. In a patient recovering from an acute myocardial infection who is taking warfarin the administration of clofibrate may

a. decrease the risk of hemorrhage

b. have no effect at all

c. increase the risk of hemorrhage

d. lower the vitamin K level

128. Alpha-tocopherol

a. causes thromboembolic diseases

b. inhibits platelet aggregation

c. is a natural oxidant (hindi daw natural)

d. is similar to alpha-interferon

129. Streptokinase and urokinase are contraindicated in the presence of

a. arteriovenous shunt

b. malignancy

c. pulmonary embolism

d. thrombosis

130. Transderm- Nitro is contraindicated in:

a. effort angina

b. elevated ocular and cerebrospinal fluid pressure

c. stable angina

d. variant angina

131. Following the administration of nitroglycerin, the change in cardiac hemodymanic is due to

a. constriction of collateral vessels

b. decreasing of myocardial oxygen consumption

c. shifting of the blood away from ischemic areas

d. venous dilation then increased venous return then rapid ventricular volume with increased wall tension;

Page 33: Prc Board Questions in Pharma 2005

132. Antitussive preparations should be used

a. in productive cough

b. in unproductive cough

c. only when hypoxia is imminent

d. in copious secrotory cough

133. All of the following drugs have antitussive properties except which one

a. codeine

b. deserpidine

c. dextromethorphan

d. morphine

134. Which one of the following phenothiazine derivatives has no antiemetic effect?

a. chlorpromazine

b. prochlorperazine

c. thioridazine

d. triflupromazine

135. Which one of the ff. gastrointestinal agents does not stimulate parietal cell function

a. acetylcholine

b. gastrin

c. histamine

d. prostaglandin

136. Which one of the following compounds has both estrogenic and antrestrogenic properties

a. chlorpromazine

b. clofibrate

c. clomiphene

d. clonidine

137. Which one of the ff/ agents inhibits spermatogenesis?

a. gelusil

b. gemcodial

c. gestodene

d. gossypol

138. The cellular metabolism of calcium is characterized by

a. calcium pumps maintaining the differential intracellular versus extracellular calcium

b. being unaffected by anticonvulsants

c. the intracellular calcium being bound only to organic phosphate

d. the intracellular concentration of calcium being 1,000-fold higher than the extracellular concentration of calcium

139. Strategy in cancer chemotherapy wherin cancer cells are held at M phase followed by administration with another CCS drugs

a. pulse therapy

b. recruitment

c. synchrony

d. rescue therapy

140. Cell cycle specific drug that acts in late S and Go phases which are used as a component of combination therapy for testicular carcinoma

a. etoposide

b. flutamide

c. leuprolide

d. mercoptopurine

141. Peptide antibiotic immunosuppressive which inhibits early stages of differentiation of T cells and blocks their activation

a. bleomycin

b. doxorubicin

c. dactinomycin

d. cyclosporine

142. Urine monoclonal antibody to CD3 antigen on the surface of human thymocytes

a. levarmisole

b. hymphocyte immune globulin

Page 34: Prc Board Questions in Pharma 2005

d. muromonab

143. Elective MAO type B inhibitor used as adjunct to parkinsonism

a. amantadine

b. selegiline

c. bromocriptine

d. haloperidol

144. Patient was diagnosed to have Huntington disease, his physician wishes to give drug, the best to give is/are.

a. tetrobenazine

b. haloperidol

c. phenothiazine

d. all of the above

145. A pro-drug that acts on platelets:

a. heparin

b. enoxaparin

c. warfarin

d. ticlopidine

145. Simvastatinand prevastatin will

a. increase clearance of IDL and LDL

b. reduce VLDL secretion from the liver

c. decrease VLDL by stimulation of lipoprotein lipase

d. reduce LDL by an unknown mechanism

146. A weak acid that compete with uric acid in renal tubule for reabsorption

a. allopurinol

b. sulfinpyrazone

c. phenocitine

d. acetazolamide

147. An antithyroid drug that inhibits organification and thyroid hormone release

a. methimazole

b. Lugol’s solution

c. I131

d. propanolol

148. A partial agonist that binds progestin androgen and glucorticoid receptors used in

a. clomiphene

b. danazol

c. mifepristone

d. relaxin

149. True of proinsulin

a. an 86- amino acid single chain polypeptide

b. cleavage and cross linking result in one chain 51 peptide insulin molecule

c. can be measured by immunoassay independently of insulin

d. all of the above

146. Streptomycinc an be used against all except

a. tuberculosis

b. tularemia

c. serratia

d. brucella

147. A 88 year old man came in because of difficulty in starting to urinate PE revealed a BP 160/100 mmHg & a slightly enlarged prostate. Which of the ff. medications is useful in treating both conditions?

a. alfuzosin

b. labetalol

c. phentolamine

d. isoproterenol

148. Which of the ff. drugs of choice in anaphylaxis associated with bronchospasm and hypotension?

a. cortisone

b. epinephrine

Page 35: Prc Board Questions in Pharma 2005

c. isoproterenol

d. phemytephrine

e. terbutolone

149. A 40-yr old patient presents with an acute hypertensive crisis. Which agent will act to block the increased blood pressure by acting at sympathetic ganglia?

a. propranolol

b. trimetaphan

c. nitroglycerin

d. digoxin

e.labetalol

150. A patient was diagnosed as suffering from chronic refractory congestive heart failure with oliguria. Which one of the ff. agents is most effective?

a. acetylcholine

b. atropine

c. dopamine

d. terbutaline

e. propranolol

151. A patient with history of bronchial asthma can be given a selective B1 adrenergic receptor blocker

a. propronolol

b. nadolol

c. timolol

d. metoprolol

e. pindolol

152. A 24 yr old female consulted because of headache palpitations and sweating BP taken

190/110mmHg.to confirm the presence of pheochromocytoma .the preferred drug would be

a. Piperocanb. Phenoxybenzaminec. Histamined. Phentolamine

153. Subsequent VMA essay done on the patient above showed elevated results medically, this patient can be managed with the following agents except.

a. phentolamine

b. propranolol

c. atenolol

d. phenylephrine

e. clonidine

154. The adrenergic receptor most likely responsible for the cardiac stimulation observed ff. intravenous injection of epinephrine is:

a. alpha 1 adrenergic receptors

b. alpha 2 adrenergic receptor

c. beta 1 adrenergic receptors

d. beta 2 adrenergic receptors

155. The nonselective beta adrenergic blocking agent that is also competitive antagonist at alpha adrenoreceptors is:

a. timolol

b. nadolol

c. pindolol

d. labetalol

e. propranolol

156. Ridorine hydrochloride is used in the treatment of:

a. parkinsons disease

b. bronchial asthma

c. premature labor

d. hypertension

157. A nonselective adrenergic agent, this binds to both beta one and beta receptors and can be used in heart block

a. epinephrine

b. ephedrine

Page 36: Prc Board Questions in Pharma 2005

c. isoproterenol

d. albuterol

158. Most common adverse effects of beta 2 selective adrenergic drugs

a. hypotension

b. hemolysis

c. tremors

d. nausea and vomiting

159. This side effect of inhalational steroids can be minimizes by washing the mouth after its use

a. gingival hyperplasia

b. oral candidiasis

c. sore throat

160. The clinical hallmarks of asthma include the ff. except;

a. coughing

b. shortness of breath

c. fever

d. wheezes

161. Considered as controllers in asthma

a. beclomethesone

b. salbutamol

c. ipratropium bromide

d. epinephrine

162. True of glucocorticoids, except:

a. decrease formation of cytokines

b. inhibit the generation of PGE2 and PGI 2

c. inhibit the production of spasmogens

d. reverse the initial bronchoconstriction in asthma

163. This mucolytic agent breaks disulfide bonds and is useful as an antidote for paracetamol

a. carbocystiene

b. acetylcysteine

c. bromhexine

164. Known as chemoattractants, action is on the later phase of asthma:

a. LTB4

b. LTC4

c. LTD4

d. all of the above

163. True of salbutamol, except;

a. given by oral and inhalational route

b. maximum effect is within 30 minutes

c. duration is 4-6 hours

d. blocks muscarinic receptors

164. These drug cause relatively little respiratory depression unless combine with alcohol

a. opiates

b. benzodiazepine

c. barbiturates

d. anesthetics

165. Preliminary studies in animals suggest that airway smooth muscle like that in the vascular is effectively relaxed by this agent. Possibly useful in pulmonary hypertension;

a. nutric oxide

b. cromakallin

c. lgE antibodies

d. nifedipine

166. Cromolyn sodium as its major action

a. block of calcium channels in lymphocytes

b. block of mediator release from most cells

c. block of phosphodiesterase in most cells and basophils

d. smooth muscle relaxation in the bronchi

Page 37: Prc Board Questions in Pharma 2005

169. A 16 yr old patient is in the emergency room. She has a heart rate 130 per min. 0f 40/min and an estimated 1-second forced expiratory volume less than 10% of end rates are audible without a stethoscope. Drugs that can dilate bronchioles during an acute asthmatic attack include all of except;

a. epinephrine

b. terbutaline

c. nedocromil

d. theophylline

169. After successful treatment of the acute attack. This patient was on follow-up treatment of her asthma. Succesful strategies currently in use are the following except.

a. avoidance of antigen exposure

b. blockade of histamine receptor

c. blockade of leukotriene

d. inhibition of phospholipase

170. True of Cromolyn sodium

a. in as aerosol that effectively inhibit both antigen and exercise induced asthma

b. in chronic use slightly reduces the overall level of bronchial reactivity

c. efective on airway smooth muscle tone and are effective in reversing bronchospasm

d. only a and b

180. One effect that theophylline, nitroglycerin, isoproterenol have in common is

a. direct stimulation of cardiac contractile force

b. tachycardia

c. increased gastric acid

d. postural hypotension

182. Theophylline acts by;

a. inhibiting phosphodiesterase

b. inhibiting cylic AMP

c. inhibiting beta receptor

d. inhibiting muscarine

183. The xanthine drugs produce this effects;

a. bronchodilation

b. decrease gastric and digestive enzymes

c. CNS depression

d. all of the above

183. Beta adrenergic agents cause;

a. stimulation of adenyl cyclase

b. inhibition of phosphodiesterase

c. antagonize adenyl cyclase

d. all of the above

184. Plasma concentration of theophylline is increased when given with

a. cimetidine

b. rifampin

c. phenytoin

185. To relieve the bronchospasm in bronchial asthma, you may give the ff., except

a. metoprolol

b. ipratropium bromide

c. isoproterenol

186. True of ipratropium bromide except

a. binds to all muscarinic receptor

b. inhibits acetylcholine mediated bronchospasm

187. This is a colorless, tasteless, odorless and non- irritating gas, a byproduct of incomplete…

a. carbon monoxide

b. sulfur dioxide

c. nitrogen oxides

188. This is a colorless, irritant gas, generated primarily by the combustion of fossil fuels;

a. carbon moxide

b. sulfur dioxide

c. nitrogen oxides

Page 38: Prc Board Questions in Pharma 2005

189. This is a brownish, irritant gas, sometimes associated with fires:

a. carbon moxide

b. sulfur dioxide

c. nitrogen oxide

189. This is a bluish irritant gas that occurs normally in the earth’s atmosphere, when absorbent of ultraviolet light;

a. carbon monoxide

b. sulfur dioxide

c. nitrogen oxides

d. ozone

190. Asthma is often exacerbated in patients exposed to this reducing agent when…air as low as 1-2 ppm. It is formed mainly form combustion agent of fossil fuels;

a. sulfur dioxide

b. rotenone

c. carbon monoxide

101. Acute exposure to this alphatic hydrocarbon solvent causes CNS depression, this has led to impairment of memory and peripheral neuropathy:

a. tetrachloroethylene

b. toluene

c. paraqual

102. This compound is a potential environmental hazard that is formed as a contaminant in the manufacture of herbicides;

a. DDT

b. dioxin

c. aldicarb

d. benzene

193. Bone marrow cell in early stage of their development appear to be most sensitive to this drug and can cause pancytoponia and aplastic anemia

a. carbon monoxide

b. carbon dioxide

c. benzene

194. Decrease incidence of EPS side effects;

a. chlorpromazine

b. triflupromazine

c. fluphenazine

d. haloperidol

e. thioridazine

195. Increase EPS but decrease tendency to produce sedation of autonomic side effects;

a. fluphenazine

b. chlorpromazine

c. thioridazine

d. mesoridazine

e. piperocetazine

True of the pharmacokinetics of antipsychotic drugs;

a. Highly lipid-soluble and protein boundb. Longer clinical duration than the half lifec. Prolonged half lifed. A and b

Antagonism of dopamine in the nigrostriatal system

a. Parkinsonismb. Galactorrheac. Amenorrhead. Hyperprolactinemiae. Psychosis

Blocking of dopamine’s tonic inhibitory effect in prolactin release in the pituitary gland causes

a. Parkinsonismb. Galactorrhea

Page 39: Prc Board Questions in Pharma 2005

c. Amenorrhead. Hyperprolactinemiae. Psychosis

Blockade of the dopamine in the mesolimbic and mesofrontal system;

a. Antipsychotic effectb. Depressionc. Hyperprolactinemiad. Amenorrheae. Galactorrhea

Effects on the medullary periventricular pathway

a. Eating disorderb. Infertility

True about clindamycin except

a. Excellent oral absorptionb. Associated with antibiotic associated enterocolitisc. Poor bone penetrationd. Prophylaxis of endocarditis in patients with valvular hearth disease for surgical procedurese. Blocks translocation reaction

Route of elimination of streptogramins

a. Fecalb. 20% urinec. Biled. A and b

Its unique binding site results in no cross resistance with other drug classes

a. Clindamycinb. Erythromycinc. Linezolidd. Synercide. Lyncomycin

Contraindications for fluoroquinolone use except

a. Elderlyb. A 30yr old patient with complicated urinary tract infectionc. Patients who are in their prepubertal staged. Patients taking theophylinee. A and b

The following are systemic acting quinolone except

a. Ciprofloxacinb. Nalidixic acidc. Gatfloxacin

Inhibits the reabsorption of uric acid by a weak acid carrier mechanism at the part of nephron;

a. Probenecidb. Coichicinec. Sulfinpyrazoned. Allopurinol

Indications for allopurinol use except;

a. Chronic tophaceous goutb. Allergic reactions to uricosuric agentsc. Recurrent renal stonesd. Normal serum uratee. Renal functional impairment

True about Aspirin;

a. Poor inhibitor of cyclooxygenase in peripheral tissuesb. Contraindicated in childrenc. Crosses BBB and placental barrier by active transportd. No anti-inflammatory effecte. All of the above

True about the toxic effects of Aspirin except;

a. Reye syndrome

Page 40: Prc Board Questions in Pharma 2005

b. Salt and water retentionc. Prolongation of bleeding timed. Antidote is N-acetylcysteinee. Allergic reaction

Therapeutic indications of diclofenac;

a. Rheumatoid arthritisb. Dysmenorrhealc. Acute MS injuryd. Postoperative paine. All of the above

Chemotherapeutic agents classified as plant alkaloid except;

a. Paclitaxelb. Teniposidec. Vincristined. NOTA

Chemotherapeutic agent classi fied as podophyflin alkaloid except

a. Teniposideb. Vp-16c. Vincristined. Etoposide

Chemotherapeutic agent classified as alkyiating agent except;

a. Busulfanb. Plicamycinc. Cyclophosphamide

Site of alkylation within DNA of alkytating agents is

a. N7 of guanineb. N2 of adeninec. N2 of cytosine

Mechanism of resistance to alkytating agents involves;

a. Increased capability to repair DNA lesionsb. Decreased permeability of cell to the alkylating drugc. Increased production of glutathioned. All of the above

Alkylating agent mostly used for CML

a. Busulfanb. Cyclophosphamidec. Dactinomycin

Function by cross linking through alkylation of DNA

a. Nitrosoureasb. Antibioticsc. Antimetabolites

Cancer chemotherapeutic agent classified as plant alkaloid

a. Vincristineb. Dactinomycinc. Bleomycin

Intermediate acting tetracyclines;

a. Demaclocyclineb. Methecyclinec. Oxytetracyclined. Chlortetracyclinee. A & B

Toxic effects of tetracycline except;

a. Renal toxicityb. Temporary discoloration of teeth in childrenc. GI distressd. Inhibits bone growthe. Teratogenic

Page 41: Prc Board Questions in Pharma 2005

Toxic effects of chloramphenicol except;

a. GIT disturbancesb. Vaginal candidiasisc. Reversible aplastic anemia (irreversible)d. Dose dependent bone marrow inhibitione. Gray baby syndrome

True about the macrolides

a. Binds to the P sites of the 30 S bacterial ribosomal subunitb. Erythromycin is the prototype agentc. Not active against Chlamydiad. Only inhibitory in actione. Consist of a single membered lactone ring

Oral preparations of erythromycin;

a. Lactobionateb. Estolate saltsc. Glucaptate

Which of the ff. is used for prophylaxis treatment of DVT;

a. Heparinb. Warparinc. Aspirin

Drug that reverses quickly the effect of oral anti-coagulant drugs;

a. Dicoumarolb. Vitamin Kc. Protamine sulfate

The ff. drugs reduces platelet aggregation by inhibiting the ADP pathway

a. Clopidogrelb. Abciximabc. Aspirin

Account for the most anti-coagulant effect of heparin except

a. Inhibition of thrombinb. Inhibition of factor VIIc. Inhibition of Xad. All of the above

The presence of tissue thromboplastin is needed to activate this pathway

a. Intrinsic pathwayb. Common pathwayc. Extrinsic pathwayd. All of the above

Fibrinolytic activators;

a. Urokinaseb. Plasminc. Ticlopidined. Vit.k

Which of the ff. clotting factors is greatly affected by heparin except;

a. Thrombinb. IXac. XIIa

Clinical effects of heparin is best monitored by;

a. Prothrombin timeb. Complete blood countc. Bleeding timed. Platelet counte. Partial thromboplastin time

The following are available Vit B12 for therapeutic, EXCEPT

a. Adenosylcobalaminb. Methylcobalamin (pero ito yung sagot sa samplex)c. Cyanocobalamin

Page 42: Prc Board Questions in Pharma 2005

d. Hydroxocobalamine. None of the above

A patient comes into a emergency room is pulse of 140 & a bp of 190/120 he is ….complaining of insomnia. Which of the ff. drugs could have caused these symptoms?

a. Metaramiolb. Ephedrinec. Phenylephrined. Amphetamine

All of the ff. are true of trimethaphan except

a. Increases BPb. Short-actingc. Binds to the cholinergic nicotinic receptord. Competes for acetylcholine receptor sites

All of the ff. are true concerning scopolamine except;

a. Plant alkaloidb. Effects similar to atropinec. Used in treatment of motion sicknessd. Competes with adrenergic receptors

The toxicity of irreversible cholinesterase inhibitors is treated with

a. Atropineb. Atropine and pralidoximec. Praidoxime

Direct acting cholinomimetics that bind and activate both the muscarinic and nicotinic receptors

a. Physostigmineb. Edrophoniumc. Metacholined. Malathione

True of physostigmine

a. Contains a tertiary ammonium groupb. Water solublec. Does not enter the CNSd. Short acting

Which of the ff. antimuscarinic drugs is used by inhalation in the treatment of bronchoconstriction

a. Dicyclomine HCIb. Methscopotamine bromidec. Ipratroium bromide

The cholinesterase inhibitor that is used in the diagnosis of myasthenia gravis is

a. Edrophonium chlorideb. Ambeneniumc. Malathion

Atropine and scopolamine will block all the effects of acetylcholine listed below except

a. Bradycardiab. Salivary secretionc. Bronchoconstrictiond. Skeletal muscle???

The efferent nerves of this system supplies only the skeletal muscles

a. Somatic nervous systemb. Autonomics nervous systemc. Central nervous systemd. All of the above

Which of the ff. statements is true about angiotensin receptor blockers

a. Inhibits angiotensin action more completely than ACE inhibitorsb. No effect on bradykinin metabolismc. Have the same side effect as ACE inhibitors except for cough andd. All of the abovee. A and b only

Page 43: Prc Board Questions in Pharma 2005

Drug that have been shown to reduce morbidity and mortality in hypertension

a. Diureticsb. Beta blockersc. Alpha blockersd. All of the abovee. A and B only

Antihypertensive agent that is useful in patients with angina;

a. Beta blockerb. Calcium channel blockerc. Hydralazined. A and b only

Antihypertensive agents that is useful in patients who also have congestion

a. Diureticsb. ACE inhibitorsc. Alpha blockersd. A and b only

Side effect related to use of ACE inhibitors include which of the ff.

a. Acute renal failureb. Cough and angioedemac. Hyperkalemiad. A and b onlye. All of the above

Vasodilator therapy leads to which of the ff. compensatory response

a. Increase renin releaseb. Tachycardiac. Orthostatic hypotensiond. All of the abovee. A and B only

Drugs with positive inotropic action includes which of the ff.

a. Captoprilb. Labetalolc. Milrinoned. All of the abovee. A and b only

Which of the ff. statements is true about digitalis glycosides

a. Inhibits Na-KATPaseb. Increases calcium concentration leading to grater systolic contractionc. Causes increase in intracellular Na concentration which leads to….d. All of the abovee. A and b only

Edema in patients with congestive heart failure is due to which mechanism

a. Decrease in renal blood flow activating the RAAS system causing retention of saltb. Increase in venous pressure causes an increase in capillary filling pressurec. Decrease in blood pressure activating the adrenergic system leading to increase arteriolar

resistance and systemic blood pressured. All of the abovee. A and b only

Beneficial effects of digitalis include which of the ff.;

a. Increase in heart rateb. Increase in cardiac outputc. Increase in production..d. All of the above